This is a main idea question because the author strongly signals the answer to this in paragraph 4, when discussing Apollinaire: “Apollinaire asserted that there were two kinds of artists: the unreflective virtuous, who relies on nature, and the cerebral structurer, who relies on understanding. Mozart could serve as prototype for the first, Beethoven for the second. As a prodigy, Picasso epitomized the second type, but, said Apollinaire, he was able to convert himself into the first. ‘Never has there been so fantastic a spectacle as the metamorphosis he underwent in becoming an artist of the first type.’” Since the author does not comment or condition Apollinaire’s strong praise of Picasso, and since the author instead goes on to support Apollinaire’s claims with the reference to The Portrait of Gertrude Stein, I know that this is the author’s understanding of what makes Picasso so great. So I’m looking for an answer that talks about how Picasso had to learn how to be more unreflective: to shift from his academic approach to painting to something more natural.
This is incorrect because the Apollinaire quote, and paragraphs 2 and 3, both argue that Picasso had to change from his childhood approach to painting, which “lacked childlikeness,” according to Picasso (paragraph 2).
The passage doesn’t talk much about the art movements of his time, so I’m immediately suspicious of B. Even if I stretched B to include the academic painting that he did as a child, and that his father did (paragraph 3), art movements aren’t mentioned at all in the key Apollinaire quote: since Apollinaire is talking about artists in general, and not artists living during a specific time with specific movements. B is therefore incorrect.
The Apollinaire quote actually implies the opposite of C, since what Apollinaire says is so remarkable about Picasso is that he transformed from a cerebral, thinking painter to one that could paint more naturally: that is, without revisions and amendments. That makes C incorrect.
D is the last answer choice standing, and luckily it is correct. It captures the heart of the Apollinaire quote: both that Picasso was special because he painted like “the unreflective virtuoso,” who would paint with spontaneity (since reflection implies planning), and, most importantly, because D says that Picasso had to learn to do this.
Without paragraph markers, this question could be hard. Luckily, the previous question involved a quote that mentions nature: the Apollinaire quote. There, “the unreflective virtuoso” that Picasso becomes “relies on nature.” Since the latter is a definition of the former, I can deduce that nature has something to do with being unreflective: with trusting impulse and instinct.
objects and scenes of the natural world. This is of course the most commonsensical understanding of the term, but the passage never talks about the natural world, so A has to be incorrect.
elementary aesthetic principles. “Elementary” is a word we often associate with schooling, and “principles” implies a very cerebral approach to painting. Therefore, I would eliminate B because it seems to resemble the kind of painter Picasso had to shift away from in order to become great: the “cerebral structurer, who relies on understanding” (paragraph 4).
intuitive artistic mastery. C is perfect, because, like the first of Apollinaire’s distinctions, C emphasizes instinct, and how that can make an artist like Picasso great.
uncontrolled creative impulses. D is the only really tempting answer choice here because it also sounds a lot like being “unreflective.” However, C has to be a better answer than D, both because C better accounts for why passion or instinct would be good for an artist—because it bestows “artistic mastery”—and because D is just far less precise than C. The example that the author gives of Picasso as a “natural” painter involves a great deal of control, in, for example, Picasso’s having Stein sit eighty times. D is therefore incorrect because it is just not as precise and comprehensive as C.
What is the author’s response to the standard story about the origin of Picasso’s genius? The author details the standard story in paragraph 2: that Picasso’s “precocious talent enabled him to surpass, without effort, all other artists in his milieu.” In other words, that story is that Picasso was so great as a child that he started off with an advantage that other artists could not catch up to. The author’s response to this account, though, is measured. Paragraph 3 begins by pointing out Picasso himself downplayed his talent as a child. This is reinforced by the author’s own opinion in that paragraph: “My own conclusion is that Picasso’s output during the first decade of his life was unusually skilled rather than frankly precocious.” So even though the author thinks Picasso was a skilled child, the author directly attacks the standard story. That attack makes sense, given that the author’s main point is that Picasso had to learn how to be more childlike in his painting: that he started off as a more academic or cerebral painter, and only gained the style he would become notorious for later in life. The right answer, then, will have to both note that Picasso rejects this standard story, and that the reasoning is that Picasso’s early art was not as promising as the standard story suggests.
Acceptance: Picasso’s early drawings are described as unusually skilled and his progress as spectacular. A is immediately wrong because the author does not accept the standard story of Picasso’s greatness.
Neutral: It is mentioned only to introduce the discussion of Picasso’s eventual virtuosity. B is also incorrect because it does not capture the effects of the author’s arguments. The author contradicts the standard story in order to make a slightly different point about Picasso, and a contradiction is never neutral.
Revisionist: It is presented as applicable only to Picasso’s earliest efforts. This is also incorrect because the author contradicts this claim as well. The quote from paragraph 3 states that what makes Picasso a prodigy is not his early efforts, but his later transformation.
Skepticism: Picasso’s earliest drawings are presumed to be not especially precocious. D has to be correct, and it definitely is. It fits my prediction well, since it both captures the author’s criticism of the standard story—that the author is “skeptical—and why: because Picasso’s earliest drawings are not precocious.
This question goes back to the same standard story that I had to reference in the last question, except this time I have to determine what it contradicts. The most immediate answer to this question comes in paragraph 2, where the author first reacts to the standard story by quoting Picasso’s own feelings about his childhood drawings: “At a youthful age I painted in a quite academic way, so literal and precise that I am shocked today […] When I was their age, I could draw like Raphael, but it has taken me a whole lifetime to learn to draw like them.” Since the author uses Picasso to contradict the standard story about Picasso’s greatness coming out when he was a child, the right answer will likely mention Picasso’s own impressions of his childhood work.
A fits my prediction, and so I’ll choose it and move on.
There is no mention of such distortion in Picasso’s reaction. However, B is mentioned in paragraph 1 as evidence of Picasso’s future greatness. That actually makes B incorrect, though, since B would be consistent with the idea that Picasso’s early art showed the promise of his later art.
The author mentions this in paragraph 3, but that is mainly so the author can speculate regarding how much Picasso’s early work resembled his father’s. More importantly, it’s not clear how these missing drawings would somehow contradict the standard story of Picasso’s greatness. Maybe if we had access to those drawings, we could tell if they resembled Picasso’s later great works or not, but I can’t see how the absence of them would contradict the assertion that Picasso’s early work hinted at his greatness. That means C is neither clearly consistent or inconsistent with the assertion mentioned in the question stem, so C has to be wrong.
D is actually like B, insofar as it is consistent with the assertion regarding Picasso’s early promise. The author never opposes the idea that “Harbingers of Picasso’s later genius” included, in his youthful work, “visual puns, outrageous caricatures, incongruous juxtapositions, and the like.” D is therefore incorrect.
This is a challenging question at first, simply because it’s asking about implied goals: goals that are not explicitly stated. However, my work for the previous questions helps me here again. I know that the author’s main idea is that Picasso had to learn how to be childlike in his work, so I need an answer that feels childlike.
I’m unsure of A. I suppose kids find objects and play with them, but found objects could be boring, or technical. I don’t feel great about calling A childlike, so I think my best bet is to not eliminate A yet, while keeping my eye out for a better answer choice.
There is definitely nothing child-like about B—what child paints with “muted colors”?—so I can eliminate it.
C feels way better than A does. The bold colors and simple faces are clearly childlike, as is the idea of using plates. I can therefore pick C and move on.
D sounds very technical, and so not very childlike at all. I can eliminate it.
It’s kind of this rather difficult passage to keep going back to the same parts of the argument. The trick to this question, though, is that it is an implicit weakener, since the passage actually argues that Picasso was not always an “unreflective virtuoso”: that Picasso had to learn to be that kind of an artist (paragraph 5). Since this goes against the passage argument, the right answer should stand out. If it doesn’t, then I can go into the answer choices and eliminate my way to the right answer, since I have a fairly good idea of what the right answer should look like. However, I also know that the author raises the idea that Picasso may have always been a childlike painter in the first paragraph: “Harbingers of Picasso’s later genius appear” in works “From 1890, when he was nine.” So I should also be on the lookout for something mentioned in that first paragraph.
This quote comes from paragraph 2, where Picasso actually implies that he started off as an artist of Apollinaire’s second type. I can therefore eliminate A.
B, though, looks correct. Exuberance and experimentation here is synonymous with the childlike playfulness that Apollinaire assigns to the artists of the first type. Since the author never contests that these were aspects of some of Picasso’s early work, B works as the right answer.
C is actually the same answer as A, since his drawing like Raphael was evidence that he painted in such an academic way. And since both A and C cannot be correct, both must be wrong.
I don’t know how D relates to a question asking about how Picasso was as an artist throughout his life, and that is in effect a question about how Picasso was as a child artist. The Stein portrait was done when Picasso was an adult, so it doesn’t really speak to how he was when he was young enough to make D correct.
This is a really tough question. It’s asking me to explain a very short quote that ends the passage, and so is not explicitly explained. Going to the quote, all I know for certain is that the quote was Picasso’s reaction to the accusation that the portrait did not look like Stein. The accusation is understandable; after all, it is supposed be a portrait, and therefore bear a likeness of a person, but Picasso’s portrait of Stein “annihilated the recognizable facial features,” and substituted “masklike features for realistic ones” (last paragraph). For Picasso’s response to make sense, he must be saying that the painting still qualifies as a portrait: as a likeness of Stein. But since it doesn’t actually resemble Stein, it must be a likeness in some other way. That should hopefully be enough to help me find an answer.
A is close, insofar as it is still claiming that the portrait portrays Stein’s likeness, but there is no way that Stein would look like that portrait in old age; the portrait is too distorted. So A is wrong.
This is incorrect because Picasso’s quote is a reaction to the finished portrait, as far as I can tell. Since it’s not clear that Picasso would continue to work on the painting when the quote was made, B is incorrect.
C fits what I’m looking for: that Picasso saw the portrait as capturing a likeness that wasn’t a literal likeness. I would therefore choose C and move on.
The passage never says anything about Picasso trying to teach viewers to look at things a certain way, so D has to be incorrect. The right answer to this question cannot come out of the blue; it has to be based on something in the passage.
The author states this very plainly in paragraph 2: “the formalist critic is concerned primarily with the work itself. Speculation on the mental processes of the author takes the critic away from the work into biography and psychology. Such studies describe the process of composition, not the structure of the thing composed.” So the right answer will either say something about the work itself, or the structure of the work (since that structure is opposed to what formalist critics avoid: biography and psychology).
This is a matter of psychology, which is what formalist critics avoid, so A is incorrect.
B is also a matter of either authorial psychology or biography, so B has to be wrong.
C is exactly what the author states formalist critics study: “the structure of the thing composed.” C has to be correct.
It is not clear what truth has to do with structure, or with the work itself. That’s more than enough to make D incorrect.
This is a broad question, so there’s no easy way to predict it. I have to test each answer choice and see if a claim from the passage depends on it.
A has to be the right answer. The entire passage is about better and worse ways to critically engage with literature. For there to be a right way to study literature—to say that critics should study the work itself, or its structure—is to imply that there are wrong ways to study literature, which is what A says.
The author would not assume B, since the author seems to agree with formalist critics who attempt to look past the author’s motivations: to get past “biography and psychology” (paragraph 2).
I can eliminate C because it simply isn’t stated in the passage, and nothing in the passage implies that good readers also need to be artists. That’s a pretty major claim to make, and so I’m sure I wouldn’t have missed it if it was mentioned in the passage.
The last paragraph says exactly the opposite: “Good literature is more than effective rhetoric applied to true ideas.” D is therefore incorrect.
Another broad question, so I still can’t predict an answer choice. This time, though, the right answer will be a claim that is supported, while the wrong answers will either not be from the passage, or will be stated without evidence supporting it.
This claim is stated in paragraph 2, but it is not supported; the author just states at as if it is obviously true. So since there is no support for A, A has to be incorrect.
The author states this in the last paragraph—“A literary work is a document and as a document can be analyzed in terms of the forces that have produced it”—but that is given as a single statement, and as obviously true. The passage is not focused on arguing it, and so I can’t say that the passage offers the most support for it. B is therefore wrong.
C, though, is argued in multiple paragraphs, with multiple arguments supporting it. The author goes so far as to explain what happens if the critic does not assume an ideal reader in paragraph 5: “Either we say that one person’s reading is as good as another’s, and thus deny the possibility of any standard reading, or else we take the lowest common denominator of the various readings that have been made—that is, we frankly move from literary criticism into social psychology.” Since the author invests in arguing C with explanation and argumentation, C has to be right.
This is stated in the last paragraph, but it is described as a fact, and as a fact, “it would be futile to deny” it; in other words, it is self-evident and therefore not in need of evidence to support it. The author therefore does not offer any evidence to support D, making D incorrect.
The author rejects biography and psychology while making the passage’s main point, so I just need an answer which focuses on that main point: that the critic should focus on the work itself.
The author repeatedly acknowledges that biography and psychology could provide some information about literature. For example, in paragraph 4, the author notes that the study of psychology is “valuable and necessary”; it is also just not what literary critics do. That makes A incorrect.
B describes what a formalist critic does, since to talk about the “author’s intention as realized” is to focus on the work: the author’s realization of intention. And since the author opposes formalist critics and biographers and psychologists, I can safely eliminate B.
C has to be wrong because the author claims that biography and psychology, while potentially valuable, miss something: miss studying the work itself, which is what formalist criticism does. So I can eliminate C as well.
D has to be right, and it is, since it echoes the author’s main idea. The author likes criticism that focuses on the product of writing, and so dislikes biography and psychology for not focusing on the product of writing: for focusing instead on the process.
Yet another broad answer choice. This time, the right answer will be a claim that resonates with the logic of the main idea, so I just need to keep the author’s argument about literary criticism focusing on the work itself in mind and I’ll have no trouble eliminating my way to the right answer.
This is incorrect because the author claims that formalist critics keep the study of the work itself distinct from the study of the people involved: the readers and authors. That makes A wrong.
B also goes against the main idea, because the critic again focuses on the work itself: not on stuff outside the work, like historical styles.
C works because the passage argument is all about what the author focuses on and ignores. C highlights one of the aspects of a work that the author ignores: the author’s intended communication, since the critic only focuses on the realized intention, which is the work itself. C is therefore correct.
D is easy to eliminate because English departments are not mentioned in the passage at all.
Again, this is a broad question, so I have to find an answer that resonates with the main idea: that is, the idea that critics should focus on the literary object itself.
At first, A catches my attention because the author uses structure interchangeably with the work itself in paragraph 2. That’s almost enough to make me choose A, but I don’t remember anything about valid or invalid structures, so my best plan of action here is to hold onto A and see if there is a better answer choice.
B contradicts paragraph 3: “instead of focusing on the varying spectrum of possible readings, the critic attempts to find a central point of reference from which to focus on the structure of the poem or novel.” In other words, if the critic has done their job right, they will not need to deal with multiple readings of a work. That makes B incorrect.
C has to be wrong because, as I’ve seen throughout this question set, critics are supposed to ignore authorial personality when studying the work itself.
D contradicts the end of paragraph 5: “We discount also such tests as the intensity of the critic’s reaction.” This has to be wrong because the only way to rate how effective or ineffective a rhetorical style is is by how it affects the reader. A good critic will therefore ignore rhetorical effect.
I can finally predict the right answer to this question. If the main idea is that critics are supposed to study the work itself, I need an answer choice which suggests that this is either bad or impossible to do.
This is stated in paragraph 3—“the formalist critic knows as well as anyone that literary works are merely potential until they are read—that is, that they are re-created in the minds of actual readers, who vary enormously in their capacities, their interest, their prejudices, their ideas”—so I know it cannot weaken the argument. The question stem, after all, by starting with “Which of the following statements, if true,” is asking me to find a choice with information that is not stated in the passage.
B is also stated in the passage—this time in the last paragraph—so B has to be wrong.
C would arguably strengthen the author’s argument, since the author wants critics to ignore authors. If authors are unreliable when it comes to their intentions, then it makes even more sense to ignore what they say and to instead pay attention to the works they wrote. C is therefore wrong.
D, though, would definitely weaken the argument. If the critic cannot ignore the social and historical contexts of a work, then the critic cannot focus exclusively on the work itself, since that critic would also have to learn about the work’s place in history, and in its society. D directly challenges the passage argument, so it must be correct.
It’s nice when a passage starts with a main idea question, since they help me to feel like I have a grasp on the passage argument. The author begins by distinguishing “literature” from “literary study” in order to argue that literary study needs to be approached as generating knowledge. The author then raises one way literary study can do this—by drawing from the sciences—only to reject that in paragraph 4. This sets up the author’s main idea, as stated at the end of that paragraph: “Literary scholarship has its own valid methods, which are not always those of the natural sciences but are nevertheless intellectual methods.” I know this is the main idea because the other clearly emphasizes it by capitalizing the beginning of it, even though it is not the beginning of a sentence. It is also what the subsidiary arguments in the passage lead to; the last paragraph simply supports that claim. So I’m looking for an answer which states that literary study is done with intellectual methods.
The author specifically does away with this idea in paragraph 1 when the passage brings up that some people have said that “one cannot and should not study Pope without trying one’s own hand at heroic couplets or an Elizabethan drama without writing a drama in blank verse.” But the author immediately argues against this idea in paragraph 2, so A cannot be correct.
This captures the passage argument through paragraph 3, but misses what goes on in paragraphs 4 and 5. The right answer to a main idea question must encompass what happens in the whole passage: the central line of argumentation. Since the passage builds off of B to make its point about methods unique to literary scholarship, B has to be wrong.
C perfectly matches my prediction from paragraph 4, so I can choose it and move on.
The author mentions this in the last paragraph, but does so in order to say that this obfuscation is unjust because literary scholarship, just like scientific scholarship, utilizes intellectual methods. Since the author uses D in order to further argue for C, D cannot be the right answer.
This is such a broad question—since the passage is about literary study, and so is entirely about what students of literature should do—that the only way to answer this question is to test each option.I. discover ways to approach literature intellectually. I know that I is mentioned in paragraph 2: “The student must translate the experience of literature into intellectual terms.” Some parts of II are also mentioned in paragraph 2, but the passage never says that students need to separate the rational from the irrational in literary works. Instead, the author acknowledges that literature has irrational elements, but encountering them is just part of studying literature, just as it is part of being a “historian of painting” or “musicologist” or “sociologist” or “anatomist.”
separate the rational from the irrational elements in literary works. Since the author isn’t
II. advocating separating the rational and irrational elements of literary works, II has to be wrong.
III. integrate the experience of literature as art and the analysis of literature as knowledge. III directly contradicts the author’s efforts in the first two paragraphs. The passage begins with the author stating that “We must first make a distinction between literature and literary study” (paragraph 1), and then says in paragraph 2 that “useful as the experience of literary creation is, the task of the student is completely distinct.” So the author would be against III, meaning that the right answer must involve just I.
A matches my prediction, so I can choose it and move on.
II goes against paragraph 2.
III goes against paragraphs 1 and 2.
And D has to be wrong because II is wrong.
Since the situation in the question stem is asking about what should happen when studying a piece of literature, I know this is actually a main idea question. I therefore need to keep in mind the author’s major points regarding literary study: that it is distinct from the creation of literature, and has its own intellectual methods. Trying to make a more specific prediction risks wasting time, so I’m best off just going through the answer choices and seeing what matches my prediction.
The author mainly links literary study to studying the work of literature itself. There is no mention of social context in the passage when it comes to studying literature, so I can safely eliminate A.
This does focus on the work itself, but something about the focus on specific words and concepts throws me off. Looking at the passage, paragraph 2 emphasizes that the study of literature should involve “a coherent scheme,” or an understanding of a text that is holistic. B is so fragmented that I’m not sure it qualifies. Still, since B does focus on the text, my best bet here is to hold onto B, but to also be ready to eliminate it once I’ve found a better answer choice.
C, like A, points to something outside of the text. Since the passage doesn’t mention anything like this historical study of texts, I can safely eliminate C.
D is luckily a better answer choice than B. Even though D contains the word “isolating,” and so seems to risk resembling B, it’s actually advocating a holistic study of the text, since that study focuses on “story elements”: elements which, definitionally, must run throughout a piece of literature (which is ultimately another word for a story conveyed in a specific medium). And though popularity seems like it is pointing to something outside of the text, it is still focused on a single text, unlike C. D is therefore the best answer.
The repeated use of “rational” in different ways definitely makes this question harder. To answer this question, I need to identify what is unclear about the quote. I have a fairly strong grasp on what a rational element in literature might look like: something which is the result of reasoning. But “irrational” and “unrational” are implied to be different by the quote, and I can’t easily deduce offhand what the difference between them would be. So I could understand this part of the passage better if I knew what an irrational element of a piece of literature looked like, or an unrational one. So my first move here is to look through the choices to see if that’s an answer choice.
Because this part of the passage is so clinical, I’m not sure that a definition would help, since definitions are pretty clinical too, and that clinicalness is part of what makes it difficult to define these terms. I’ll keep A around for now, but will be ready to eliminate it when a better answer choice comes along.
B works way better than A. If part of the problem with the quote is that it is difficult to conceive of what an irrational or unrational element in literature looks like, I could better picture such an element with an example than with a definition. B also helps because it hones in on the more problematic term. I know what irrational means, but I would assume that unrational means the same thing. Since the passage implies that the two are distinct terms, seeing an example of the term I’m unfamiliar with can help me to better understand the statement in the question stem. I can therefore eliminate A and choose B.
The author is talking about the work, and not the creative process, so I can immediately eliminate C.
Realism isn’t an obvious aspect of the quoted section, so D is already wrong on that count. More problematically, the challenging part of the quote is understanding what the author means by unrational, which D also doesn’t mention. D therefore has to be wrong.
Most of the passage is devoted to just arguing that there even is something distinct called literary study. So if the author is ever going to talk about why that study is important, it would have to take place in the last paragraph. There, the author states “Only a very narrow conception of truth can exclude the achievements of the humanities form the realm of knowledge. Long before modern scientific development, philosophy, history, jurisprudence, theology, and even philology had worked out valid methods of knowing. Their achievements may have become obscured by the theoretical and practical triumphs of the modern physical sciences, but they are nevertheless real and permanent.” I can sense in the author’s words here a desire to have people respect the knowledge that literary study and the humanities more broadly can produce, especially when the author states that the opposite view—that knowledge comes mainly from science—comes from a “very narrow conception of truth.” To use that intensifier “very” is to imply here that the conception of truth is too narrow: that it needs to be broadened to respect the knowledge that literary study, as its own discipline can bring.
The author explicitly distinguishes literary studies from the sciences in paragraph 4, so A can’t be right.
The author also distinguishes literary study from “literary creation” in paragraph 2, so B is wrong.
Truth is a loaded phrase; I should always be careful to not assume that a passage is talking about truth unless it explicitly mentions the term. Since I don’t see anything about truth in the passage, C is incorrect.
Notice that while D resembles C, it is far more tempered; rather than claiming to talk about truth, it instead just mentions “a type of knowledge.” D works because it is in line with the author’s main idea—if “Literary scholarship has its own valid methods, which are not always those of the natural sciences” (paragraph 4), then it would produce knowledge unique to it, and so unavailable to other disciplines—and the last paragraph, insofar as D stands up for the “achievements of the humanities” by implying that literary study generates knowledge. That makes D correct.
To answer this question, I need to understand the effect that new information has on specific claims from the passage. Two parts of the situation stand out to me: 1) that the situation mentions contemporary literary studies and traditional literary criticism, perhaps implying that these are two distinct disciplines rather than the same discipline at different points in time, and 2) that contemporary literary studies involves “systematic analysis,” which feels very scientific, or is at least something we associate with the sciences, though it is obviously not exclusive to them. A quick scan of the passage for mentions of analysis and literary study brings me to paragraph 4: “There is, no doubt, a large field in which the methodologies of science and literary study contact or even overlap. Such fundamental methods as induction and deduction, analysis, synthesis, and comparison are common to all types of systematic knowledge.” So perhaps the answer has something to do with commonalities between literary studies and scientific study.I. It is consistent with the assertion that the transfer of the methods of the natural sciences to art has not fulfilled expectations. Scanning the options, I is incorrect because the situation in the question stem seems to suggest that “contemporary literary studies” have met with some success by finding something that “traditional literary criticism” missed. If anything, the situation attests to how the transfer of methods from the natural sciences to literary studies has helped.
II. It constitutes evidence for the assertion that there is a field in which the methodologies of science and literary study overlap. II, though, looks good, since that is just a restatement of the quote from paragraph 4.
III. It is inconsistent with the assertion that philosophy, history, and theology have worked out valid methods of knowing what can be modified to apply to the humanities. Finally, the situation in the question stem is way narrower than III is. I don’t see how a discussion involving literary study would affect the broad claims about philosophy, history, or theology mentioned in III, so III is incorrect.
I goes against the information in the question stem, so A is wrong.
II matches my prediction, so B is correct.
I is wrong, so that’s enough to make C incorrect.
I know III is wrong because it is so broad that the information in the question stem can’t suggest it, so D is wrong.
This question is asking me to sense out what part of the passage resonates with claims from outside of the passage. The best way to do this is to make sure I understand what this new information is arguing in terms of the passage. The two quotes seem to be suggesting that literature is unique: is unlike the other disciplines mentioned. That directly reminds me of the main idea as stated in paragraph 4: “Literary scholarship has its own valid methods.” This claim is in service to saying that literary study is distinct: explicitly from the sciences, but implicitly from other disciplines, including those in the humanities. So I’m looking for an answer choice which says something about literature being its own distinct field of study.
A actually suggests that literature might need to draw on sociology or politics, which is close enough to suggesting that literature could be a substitute for either to make A wrong.
B fits my prediction. It is a claim argued in the passage, and seems in line with the quote from the author above, since to say that literature is not sociology or politics or philosophy is to begin to argue that literature, and therefore literary study, are unique. I would choose B and move on.
I would eliminate C because the passage doesn’t really distinguish between artistic pursuits and other pursuits except in one specific instance: distinguishing literature from literary study. Because the quote from the question stem compares literature to a variety of disciplines, the quote doesn’t really focus on literature as art; it just focuses on literature as different from sociology, politics, or philosophy. Since the question stem is not more specific regarding why literature is distinct from these disciplines, that question stem is not similar enough to C to make C correct.
D has to be wrong because science isn’t mentioned in the question stem, so D is not in agreement with the quote from the question stem.
While the passage discusses film placement effectiveness in multiple parts of the passage, this question is asking about what is “most effective”: in other words, what method of film placement works best, among a set of options. For that, I need to go to paragraph 2, which mentions that “fees are usually based on a hierarchy of product treatments. Simple visual exposure is the least expensive, verbal mention is moderately priced, and character usage is the most costly.” It only stands to reason that the more expensive the option, the more effective the film placement, so the right answer should involve character usage: a character using the item.
A is meant to be attractive because it mentions a major character, but a reference is not on-screen usage, so A has to be wrong.
B lacks any mention of a character, so I can eliminate it.
This sounds like “Simple visual exposure,” which is the least expensive option, and therefore the least effective of the options mentioned. C is therefore wrong.
D is meant to look like a bad answer choice because it involves a “minor character,” but this sounds like character usage, since a character is using the product on screen. D is therefore the right answer.
This is a great question because it really tests if I can to set aside whatever personal biases I might have and just rely on what the passage says. People can have all sorts of strong opinions about brand placement: so much so that it can be easy to miss that the author actually has a pretty good opinion of it. The author describes the benefits of brand placement for marketers in paragraph 2, but this attitude is clearest insofar as the author takes the time to defend brand placement against the critics mentioned in paragraph 5. And since the author actually doesn’t really focus on the downsides to brand placement, I know I am looking for an answer which suggests that the author has a positive attitude towards brand placement.
A is what I’m looking for, so I can choose it and move on.
This is probably the most attractive answer choice after A, and it’s easy to imagine someone feeling like they have to make a choice between A and B. But B is wrong because the moment the author argues against the critics from paragraph 5 in the last paragraph, the author is taking a stand, and so is no longer neutral.
The author mentions some people who mistrust brand placement—older audience members and some critics—but the author takes the time to address or explain their concerns, so C has to be wrong.
D is wrong because it’s an even more extreme version of C.
Though this seems like a broad question, the author actually only talks about the effects of brand placement in the last two paragraphs. Critics think that brand placement manipulates people into buying products, but in paragraph 6 the author says otherwise: “The audience awareness of the persuasive intention of brand props in fact promotes skepticism and resistance to their pervasive influence.” In other words, brand placement actually helps people to learn that brands are trying to manipulate them, which in turn makes people less susceptible to that manipulation. That has to be enough for me to find the right answer.
The passage only mentions that young people benefit psychologically from brand recognition in paragraph 3. Since that paragraph implies that older audience members don’t experience the same benefits, A can’t count as “the general effect of brand placement.”
This also goes against paragraph 3, and specifically the older audience members mentioned there: “They report feeling resistance, discomfort, and concern.” Those are terms which suggest that they do not accept those displays as legitimate, making B wrong.
C matches my prediction from the last paragraph, so I would choose it and move on.
I don’t recall the passage saying anything about how brand placement affects film attendance, so D has to be wrong.
This weakener focuses on the “research” described, and the only research that comes to mind is in paragraph 3, though the findings are given in paragraph 4. A weakener of this sort, though, can either weaken the findings (more common) or point out a problem with the research. I know that the intent of the research is stated in paragraph 3: “to clarify the way audiences interpret brand placements in relation to movies, movie viewing, and social experience.” The results themselves focus on how people report feeling. Based on that, the author states that the research shows that “The success of brand props in exerting persuasive effects is not a matter of what the placements do to movie audiences but what the audiences do with them” (paragraph 4) But what’s weird about this conclusion is that the research only asked people what they felt about brand placement, and doesn’t say anything about what the participants did: if they bought what they saw or not, for example. So while I will be generally looking for any answer choice which weakens the passage argument about research—since there are many different ways to do that work—I will more specifically be looking for an answer choice which suggests that the study failed to account for how people acted, by instead focusing on what people said they felt.
The passage doesn’t mention what these participants were told, so there can’t be a problem with what the participants were told. A has to be wrong.
I can’t quite see what’s wrong with B. The passage doesn’t otherwise discuss socioeconomic status or belief systems, and it’s not clear to me how either of those would be more comprehensive than using age, or better suited for the study subject. Since I cannot clearly see how B could weaken the research mentioned in the passage, B has to be wrong.
C is probably meant to trick me into thinking that it’s actually saying that the participants didn’t give their actual opinions, but as it is actually stated, I can’t see why C would affect the argument about the research. The research doesn’t require that the participants form independent opinions about brand placement; all that matters is that the participants stated opinions that they held, not how they reached those opinions. That makes C incorrect.
This is the last choice left, and luckily it matches my prediction. The author’s conclusion about the study makes a claim about what audiences do, but the study chooses to focus on participant opinions about brand placement, which does not necessarily correlate with what they actually do. D has to be correct.
The work I did on an earlier question helps me with this one. I know that the reference to young audiences is meant to make me think of the studies mentioned in paragraph 3: “younger informants consider the appearance of name-brand products in the story an acceptable and expected part of the movie-going experience and not a change. For them, encounters with familiar products are associated with feelings of belonging, comfort, and security.” Since brand placement is associated with such positive feelings in younger people, the question stem is suggesting that Voom shoes should sell well, when they aren’t. But the key part of that quote from paragraph 3 to pay attention to is the claim “encounters with familiar products are associated with feelings of belonging, comfort, and security.” If familiarity is what makes young people feel good about brand placement, then those good feelings might not be counted on for unfamiliar products. That feels pretty tenuous to me, but that’s probably about as good of a prediction as I can get for this question.
A matches what I’m looking for, so I’m inclined to choose it, but I’m uneasy about this question, so I would eliminate the other answer choices as well.
There’s no way that I can know B from the question stem, and the studies cited, as we determined in the previous question, don’t say anything about the actual buying behaviors of young people. So B has to be wrong.
Young people aren’t mentioned in the passage as being concerned about the social implications of brand placement, so C is incorrect.
D is also not mentioned in the passage discussion of the studies. More broadly, the passage doesn’t really ever say that the story can get in the way of brand placement effects, so I just don’t have enough information to choose D. Therefore, D has to be wrong.
This is a tough question that asks me to use tone, intent, and what is included and left out of the description of the studies to deduce who may have conducted those studies. This can be a difficult question type to predict, since many different groups of people might have the same reasons to run a study, so my best bet here is to just keep the studies in mind (which shouldn’t be hard, since this same study has been referenced so many times in this question set), and test each answer choice against paragraph 3.
The studies ask about feelings, and it’s hard to imagine that a regulatory agency would be interested in feelings. That’s enough of a disconnect to make A wrong.
B makes a lot of sense. Advertisers would be interested in how people felt about brand placement to help advertisers determine if brand placement was worth using, and the findings are positive in a way that would make advertisers happy. But since I don’t have a good prediction for this question, and it could be wrong for reasons that I’m just not thinking of at the moment, I will hold onto B and see if I can eliminate the other answer choices.
I don’t see anything about consumers having their rights threatened in the studies, so C has to be wrong.
D might be the only other attractive answer choice in this set, since I can imagine that a psychologist would be interested in people’s feelings. However, I’m not sure why an experimental psychologist would run studies as paragraph 3 describes them, since those studies don’t involve experiments. That’s enough to make D incorrect, and B the right answer.
Another weakener, though this one is asking about brand placement. I know that the author’s defense of brand placement comes in paragraph 4—“The meaning and relevance of brands encountered in films are not simply transmitted to viewers; rather, viewers interpret these props as part of their own everyday life, and in reflecting the viewers’ past, present, and anticipated experiences, the props come to life”—and the last paragraph: “moviegoers are active interpreters, not passive receivers, of movie presentations of brands […] The audience awareness of the persuasive intention of brand props in fact promotes skepticism and resistance to their persuasive influence. For various reasons, moviegoers may allow themselves certain indulgences in some buying situations, but they are not deceived into rushing out of movie houses to buy everything shown in a movie.” Read together, the defenses of brand placement amount to saying that it is not as directly effective as those who attack brand placement suggest. So I’m looking for an answer choice which implies that brand placement is actually quite successful: that it affects consumer behavior.
A doesn’t involve any brand placement, and so wouldn’t attack the arguments about brand placement. A is therefore incorrect.
B does involve brand placement, but B is actually in line with the last paragraph of the passage, which argues that moviegoers are pretty savvy when it comes to brand placement. The attempt to educate moviegoers doesn’t work, B implies, because the moviegoers are actually pretty well-educated enough already regarding brand placement. Since B can be read as in line with the arguments in favor of brand placement because it is ineffective, it can’t weaken it.
C isn’t what I was expecting, and I almost eliminated it simply because it mentions sales of products declining. But my prediction was just that the right answer had to suggest that brand placement was actually effective, and C actually does provide an example of brand placement affecting people’s brand purchasing habits. C specifically weakens a claim from the last paragraph: “moviegoers are active interpreters, not passive receivers, of movie presentations of brands. Furthermore, they are not influenced uniformly by these encounters, a finding that suggests a mitigating effect by other factors—e.g., perceived needs, self-image, past experiences, plot context, and demographic group—on any induced tendency to purchase the product.” C lacks any mention of those factors, but registers a change in consumer behavior. So if C is true, those other factors might not mitigate brand placement. C is therefore correct, but given how unexpected C was, I would eliminate D just to be sure.
D doesn’t involve brand placement. A promotional film is an overt act of advertisement that does not function as brand placement does, so D would not clearly affect the passage arguments defending brand placement. D is therefore wrong.
This is a strengthener, but one that the answer choices tell me I can break down into two actions: first, to determine if the answer choice will strengthen the passage or not, and then why. I know that the passage author argues that pre-Civil War Americans actually really liked to read. If it was the case that American publishers could get and publish European novels at a higher profit than American ones, that has to support the author’s argument, since it suggests that there was an appetite for books in America. That means that I can eliminate C and D; I should move onto the answer choices after this rather than try to predict the second half of the right answer to save time.
A works really well as the correct answer. If European novels could be published for more money, and if half of the book reviews from 1840 to 1860 were of American novels, that means that the appetite for American novels must have been high enough to encourage publishers to publish American novels, even though it might have been less profitable to do so. I would check B, just to be sure it isn’t correct, but I’m strongly inclined to choose A and move on.
This is actually another attractive answer choice, since literary nationalism would deter American publishers from publishing European novels. If European novels were more profitable in America, that should imply that literary nationalism was not a viable force during this period. However, B is incorrect because the author never talks about literary nationalism, so there is no passage author view on the subject to reinforce. B is therefore incorrect.
I can eliminate C because it should start with “Yes,” not “No.”
D also has to be incorrect for the same reason.
I know that the right answer to this question will not be accompanied by some historical fact to support the claim. Similarly, I know that the three wrong answers will be accompanied by such historical facts. It’s usually easier to eliminate the wrong answers than it is to identify the right one for this kind of question: since it is almost always easier to identify the presence of something than it is the absence of something.
This is stated at the end of the passage. That last paragraph does mention two historical novels—The Scarlet Letter and Moby Dick—but those two are actually mentioned as evidence that America was “a nation of novel readers.” It’s also not clear that a reference to two books would constitute “historical research.” Since there is no other support for A in the passage, A has to be correct.
This is clearly supported by the historical survey of American magazine reviews in paragraph 3, so B has to be wrong.
This is clearly supported by the survey of reviews of American novels in paragraph 4, so C is incorrect.
This is supported in paragraph 3, when the author describes how American magazines grew in number from 125 in 1825 to more than 600 in 1860. That makes D incorrect.
This question is asking me to confirm the function of a piece of text: to understand what it does in the larger argument. I know that paragraph 2 is focused on refuting the claims made in paragraph 1 regarding the standard description of the state of the novel and the American reading public before the Civil War. But since this kind of question can get quite technical, I should go back and check what, specifically, the quote from the question stem is doing. Luckily, the paragraph is formatted in such a way that I can easily determine the function of the quote. The author uses a “first, second, third” approach to structuring the paragraph, which tells me that each time the author enumerates a claim, the paragraph is making a discreet point, and everything that follows that enumerated claim will either explain or support that claim. So, since the preceding sentence to the quote in the question stem reads “For one thing, expressed hostility to fiction was no less strong in England than in America; much of what Americans wrote and said about novels was derived from sources within the British Isles,” I know that the quote in the question stem is meant to offer support for that claim. That should be enough for me to recognize the right answer.
The phrasing in paragraph 2 implies that the quote from the question stem is not introducing a new point. I definitely know this because the reference to Scottish common sense philosophy is made in paragraph 1. In a sense, the quote from paragraph 2 is a point that is developed from earlier in the passage; that makes A incorrect.
B matches my prediction, and so I can choose it and move on.
The passage never says that American hostility to the novel started abroad. The passage only states that “much of what Americans wrote and said about novels was derived from sources written in the British Isles,” but that could have been because Americans were already predisposed to dislike novels, and just found the words to state their dislike from British publications. Since C isn’t a point that is clearly made in the passage, it can’t be correct.
The first mention of Scottish common sense philosophy does this, but it’s clear that this second mention is meant to highlight that it is unlikely that Americans would have uniquely disliked novels because of Scottish common sense philosophy, given that Scottish common sense philosophy would have understandably been strong in Scotland and the British Isles. That makes D incorrect.
To answer this question, I need to determine what part of the passage some new information resonates with. Most of the passage is about how America was actually a good place for novels, so I know that if I’m going to find something in the passage that is similar to the information in the question stem, I need to look in the first paragraph: where the author describes the reasons why people saw pre-Civil War America as hostile to the novel. The first point about “Scottish common sense philosophy” doesn’t seem to have much to do with the question stem, nor does the second point, since “sparseness” means something more like “plainness,” which I don’t see in the question stem. However, the end of the paragraph does include a word that is in the question stem: “The would-be American novelist before the Civil War was drawn, or forced, toward a literary form better suited to American imaginative space: the romance, created in an ambience of isolation, alienation, defiance, and apology that left its traces in the work.” So I’m looking for an answer choice that mentions that Americans wrote romances.
This is the second point, but it’s one I determined doesn’t clearly relate to the situation in the question stem, since the “sparseness of American social life” isn’t clearly the same thing as “widespread breakdown of social cohesiveness.” So I can eliminate A.
B mentions the literary romance, and I know I’m talking about the nineteenth century because that is a pre-Civil War era, so B makes a lot of sense. However, it’s possible that I might not have known about the nineteenth-century part, in which case I would hold onto B and eliminate the other answer choices.
There is no way that C could be correct, because those reviews talk about the novel flourishing in America, while the situation in the question stem talks about the “death” of the novel. C’s almost the exact opposite of what the right answer needs to contain.
There’s almost nothing in the passage about D. The second paragraph mentions that British attitudes towards the novel were probably influenced by Scottish common sense philosophy, which the first paragraph implies helped “create an atmosphere hostile to fiction,” but that’s not enough to suggest that the novel was in trouble in England, so D has to be incorrect, and B has to be correct.
The time period mentioned in the question stem brings me to the arguments made in paragraphs 3 and 4, but 4 is the only one that brings up the contents of American periodicals of the time. I know from my initial reading of the passage that this is a pro-novel section of the passage; it’s arguing that these periodicals say good things about how popular the novel was in America at the time. The paragraph says that research shows “more than two thousand reviews of eight hundred separate novels, about half of them American in origin.” I also know that “American novels were received warmly by reviewers.” Before going to the answer choices, I have to keep in mind that this is a LEAST question, so the right answer will not be consistent with the claims made in paragraph 4, while the wrong answers will. And since it is usually easier to see what is consistent than it is what isn’t, I should prioritize eliminating the wrong answers over trying to recognize the right one.
A feels resonant with the passage, both because paragraph 4 argues that there was a lot of reviews of novels, and because so many of those reviews were positive. It also helps to know that paragraph 4 states that “those on magazine staffs” “set themselves up as guardians of critical integrity” when they “complained about indiscriminate puffery among reviewers.” “Puffery” here means that reviews were too positive and not critically engaging enough, so there’s at least something in the passage which suggest that some people were upset with the published reviews. All that makes A incorrect.
The right answer to a question like this could either be random, or go directly against the passage arguments concerning the information mentioned in the question stem. In this case, B goes directly against the passage argument, which is that Americans were not hostile to their own novels. B is therefore clearly correct, since there is no way that the author would include information like B when trying to make the passage argument.
I know from paragraph 2 that “much of what Americans wrote and said about novels was derived from sources written in the British Isles,” so C would be in keeping with the publication practices of the time. That makes C incorrect.
Paragraph 1 mentions that Scottish common sense philosophy was present in American culture at the time, and does not deny this presence in paragraph 2 when the author attacks the perspective laid out in paragraph 1. So there’s no reason to suspect that D would not appear in an American publication of the time. D is therefore incorrect.
The “in fact” that introduces the quote in the question stem makes the quote an emphasized statement. Since there are no contrast keywords in this part of the paragraph, I know that this has to be evidence of the author’s argument in that passage: that it somehow serves as proof that Americans liked their novels.
A matches my prediction: that those workers serve as evidence of an attitude that was friendly to American novels. I can choose A and move on.
There is nothing ion the phrasing of the quote which suggests that the author finds something questionable about those workers, so I can eliminate B.
C is a more intense version of B, but it’s wrong for the same reasons B is wrong: because the author does not clearly suggest that there is something wrong with how these magazine staffers approached the novel. The author uses those staffers as proof that American novel reviews were often positive. Whether or not those reviews were too positive or not is beyond the scope of the passage.
D is an even more intense version of B and C, and so is wrong for the same reasons they are.
This is an uncommon question type that asks me to figure out what new information would help me to determine if the author’s findings are correct. This is therefore neither a strengthener or a weakener; I’m only trying to choose options which will help me determine if the author is right or not regarding how friendly America was to its novels.I. The degree to which reviewers’ conclusions about the relative popularity of different novels were accurate I makes a lot of sense, since if the reviewers are incorrect about how popular novels are, the reviews would be less useful as an indicator of how popular the novel was in America.
II. The extent to which the length of the reviews a book received correlated with its popularity I’m suspicious of II, though, even though the fourth paragraph mentions the length of reviews. That mention is largely there to suggest that people were writing all kinds of reviews; the author never suggests that longer reviews were made for more popular books, or vice versa. So I can safely eliminate II.
III. The relative popularity of conventional novels and romances among American readers III makes sense, though, since the author mentions that people traditionally think the romance was the most popular genre in America. If conventional novels are as popular as romances, that supports the author’s argument, while if they were less, that might hurt it. So I’m looking for a choice that mentions options I and III.
I predicted I is correct, but III is too, so A can’t be correct.
B has the same problem that A does; it’s missing a correct option.
II is wrong because it distorts the passage logic.
D matches my prediction, so I can choose it and move on.
This is a simple main idea question made more difficult by a passage that ranges so widely that it can feel unfocused. The arguments about Confucius and the effectiveness of li leads into a discussion of performative utterances: into words and phrases that are acts themselves, as opposed to descriptions of action. All that seems to sum up in the last paragraph: “What we have come to see, in our own way, is how vast is the area of human existence in which the substance of that existence is the ceremony. Promises, commitments, excuses, pleas, compliments, pacts—these and so much more are ceremonies or they are nothing.” So I expect that the right answer will talk about how important ceremony is in our lives.
A is meant to be tempting because it echoes the last sentence I read in the passage. But it also distorts it slightly, since the author’s point is not that they are verbal conventions, but that they are ways that ceremony is important in our lives. That’s enough of a divergence from the passage for me to eliminate A.
Li is certainly the most provocative aspect of the passage, which makes this an attractive answer choice. However, the Austin arguments aren’t there to clearly support an argument about li. Instead, the argument about li and Austin are there to help support an argument about how much of a role ceremony plays in our lives. Since B only covers a part of the passage argument, then, it has to be incorrect.
The passage does not focus on honesty, so I can eliminate C.
D is left, and it echoes the author’s summation of the passage argument in the last paragraph, so I can feel confident that this is the right answer.
The definition of li in this passage is actually pretty slippery, which is unusual for passages. Luckily, during my initial reading of the passage, I noted that the third paragraph talks using li in the abstract, which helps to contextualize the next paragraph’s actual example of using li: “The force of coercion is manifest and tangible, whereas the vast (and sacred) forces at work in li are invisible and intangible. Li works through spontaneous coordination rooted in reverent dignity. The perfection in Holy Rite is esthetic as well as spiritual.” That’s a really difficult argument to understand, but the example of the teacher asking a student “in an appropriate and polite (ritual) formula” to fetch a book in the next paragraph helps to clarify: li is magic insofar as treating people with respect and dignity works as well as “commands, threats, regulations, punishments, and force” (paragraph 3). So the answer should say that li works because it respects others.
A matches my prediction, but the definition of li is so slippery that I’m going to eliminate the other answer choices before choosing A.
B has to be wrong because “magical powers” isn’t as precise as the passage is. The example from paragraph 4 makes it clear that li is not about actual magical powers, but is rather about an effect so special that it could be considered magical. Since B doesn’t clarify whether it is talking about actual or metaphorical magic, B has to be incorrect.
This overextends passage example of the teacher. It is true that the teacher does not have to expend effort to get the wanted book, but the example isn’t about how much or little effort one expends when using li. Nor does the passage argue that li is best used when you don’t want to expend effort. The author is instead pointing to the harmony that li can achieve, which other means of achieving one’s goals cannot. So C is incorrect.
D is also mentioned in the passage—this time in paragraph 3—but the author does not suggest that this is why one should use li. If D were correct, then the author would have taken the opportunity presented in paragraph 3 to condemn “commands, threats, regulations, punishments, and force.” Instead, though, the author is restrained by just mentioning that the contrast between the two is not that one is better than the other, but rather that one is “manifest and tangible,” while the other is “invisible and intangible.” Since the author does suggest that force is risky, D has to be incorrect.
The first time that the author uses the word “magical” is in the first paragraph: “Confucius saw that the truly, distinctively human powers have, characteristically, a magical quality.” The next sentence has to explain what “magical” means, either implicitly or explicitly: “His task, therefore, required, in effect, that he reveal what is already so familiar and universal as to be unnoticed.” So magic refers to what goes unnoticed. Given that the author’s argument is about how so many aspects of our everyday lives are secretly ceremonial in nature, I am looking for an answer choice which brings up this hidden character.
A works because it also describes a power “behind”—and therefore hidden by—ordinary social conventions. This definition of magic is in line with the arguments about Austin later in the passage: about how some words are secretly also actions. That’s enough to make A correct.
The author doesn’t mention actual supernatural powers in the passage discussion of magic in that first paragraph, so I can eliminate B.
While Confucius seems to have come up with the idea, the passage never says that one can only learn li through Confucius. That’s a strong enough claim that I would have remembered such an argument if it were made in the passage. C is therefore incorrect.
D distorts the example of the teacher, who uses words to get others to fetch a book. It’s not clear that the act is done through “words alone”; it would be more clearly so if no students were involved, since the situation required both words and students to work. And since li emphasizes the harmonious interaction with the student, it’s unlikely that Confucius would say that a person’s words, by themselves, accomplished the task of fetching the book. So I can eliminate D as well.
Performative utterances are described in paragraph 5: “statements we make which function somewhat like the ‘operative’ clause in a legal instrument.’ That’s hard to understand, so the author clarifies this in the next sentence: “They are the very execution of the act itself.” Even that can be difficult to understand, so the next sentence gives an example of “I give and bequeath my watch to my brother,” which is an act of bequeathing rather than a description of an action. So the right options will also be actions.I. Please help me. I is a request for an action, but the request is not an action in and of itself; if I say it, I’m not doing the act that the phrase describes.
II. I promise to help. II, though, works as a performative utterance, since it is both a statement and an action (the action of promising).
III. May I help you? III functionally resembles I, insofar as it is a question and therefore a prompt for action, rather than an action in and of itself. I’m definitely not doing what III describes. So II is the only correct option.
I is incorrect because the actual saying is not the action that it says.
Here, promising to help is an action: the action of promising. So B is correct.
I is incorrect, so C is incorrect.
III is also not an example of an action which is also a phrase. Asking someone if I can help them is not actually helping them. So D is incorrect.
This refers to paragraph 3 again, where the author notes that “The perfection in Holy Rite is esthetic as well as spiritual” because “Li works through spontaneous coordination rooted in reverent dignity.” So I’m looking for an answer choice which discusses dignity or respect.
A fits, so I can choose it and move on.
The passage does not describe any such laws, so B is easy to eliminate.
C is incorrect because Confucius talks about all of humanity in terms of li, and not just refined society. C is too restrictive to be correct.
D is wrong for the same reason C is: because it is too restrictive. Li is an everyday occurrence, according to Confucius; we just don’t notice when it’s happening. And so since everyone is capable of it, limiting it to holy persons would distort the passage argument.
Austin’s argument is meant to help reinforce the author’s claims about li: that sometimes, the speaking of something, or the manner in which we do something, is efficacious in and of itself. Austin is specifically talking about language, though—as are the answer choices—so Austin must be talking about how some uses of language are acts in and of themselves.
A directly contradicts Austin’s claims about “how much the ritual word is itself the critical act rather than a report of, or stimulus to, action” (paragraph 5). So A can’t be right.
This choice attempts to combine Austin’s claims about performative utterances and Confucius’s claims about how li exposes something which is concealed in human interactions. However, Austin doesn’t talk about lying, or obscuring facts, so I can eliminate B.
C matches my prediction, and so it is correct. Note that this is a fairly weak statement—it only says that language and actions “can” be inseparable, not that they “must” be—which makes C easier to justify.
This is incorrect because Austin does not talk about social conventions. Austin is first and foremost focused on performative utterances; Confucius is the one who talks about social conventions, as does the author. Since D is not focused on Austin, it has to be incorrect.
The part of this question stem which caught my eye was how the businessman did “NOT fit comfortably into a self-governing society.” That is a phrase from the third paragraph of the passage, which criticizes Crèvecoeur: “He did not see what many Americans of his generation did, that a purely economic person would be as unsuited to a self-governing society as would the rank-bound subject of traditional regimes.” While that tells me that the right answer will not be Crèvecoeur, the quote does not clearly suggest who the example in the question stem would support. Luckily, the next sentence clarifies that by mentioning someone who did notice that an economic person might not fit into a self-governing society: “Fortunately, another Frenchman, Alexis de Tocqueville, who visited the United States in the 1830s, gave a much more adequate view.” That sentence is enough to tell me that de Tocqueville will be the correct answer.
Paragraph 3 specifically claims that Crèvecoeur failed to account for such an example, so A can’t be right.
B matches my prediction, so I can eliminate it.
Enlightenment thinkers are mentioned in the passage, but they are implicitly contrasted against de Tocqueville in the fourth paragraph. Worse still, Crèvecoeur is described as schooled by French Enlightenment in the second paragraph. Together, that implies that C is actually the same answer choice as A, and so is wrong for the same reason.
French revolutionaries are alluded to in paragraph 4, where the author mentions that de Tocqueville was wary of the promises of the Enlightenment because of the French Revolution. However, this is a passing reference, and one that doesn’t tell me much about what French revolutionaries believed or did. Since I don’t know the view of French revolutionaries, D has to be incorrect.
This question is really asking me to explain the differences between de Tocqueville and Crèvecoeur: or rather, what differences the author points out. Luckily, the author is pretty explicit about this at the end of paragraph 4: “He added to Crèvecoeur’s earlier sketch a more penetrating and complex understanding of the new society, informed by republican convictions and a deep sensitivity to the place of religion in human life.” To say he added to Crèvecoeur with his republican convictions is say that those convictions helped him to be a better observer than Crèvecoeur, so the answer has to refer to these republican convictions somehow.
This is a claim associated with both Crèvecoeur and de Tocqueville, so it can’t be something which made de Tocqueville more accurate than Crèvecoeur.
B matches my prediction, so it is correct.
Self-interest is described as an Enlightenment principle in paragraph 2, and is therefore more strongly associated with Crèvecoeur than de Tocqueville. That makes C incorrect.
The passage only mentions that Crèvecoeur had an eighteenth century education. Since I don’t know about when de Tocqueville was educated, I can safely eliminate D.
Since this talks about the specifics of an argument from the passage, I should double check it. Mores are brought up at the end of the passage, where the author notes that “He spoke of mores somewhat loosely, defining them variously as ‘habits of the heart’; notions, opinions, and ideas that ‘shape mental habits’; and ‘the sum of moral and intellectual dispositions of men in society.’” That’s a lot of ambiguity for a definition, so to feel more confident before heading in to the answer choices, it makes sense to scan the rest of the paragraph for anything else it says about mores. Earlier, the author distinguishes mores from laws when the author says “mores […] more than the laws] have contributed to “the maintenance of a democratic republic,” which tells me that mores are unofficial. Hopefully that will be enough for me to find an answer.
I can’t easily tell how anything in the list at the end of the passage relates to social rank, which is enough for me to set A aside for now.
Philosophy feels far too rigid and official to qualify as “mores.” When de Tocqueville says that mores involve “the sum of moral and intellectual dispositions of men in society,” I can’t assume that all the men in American society were familiar with French philosophy. That makes B incorrect.
C, though, is an attractive answer choice. Paragraph 4 ends with the claim that de Tocqueville had “a deep sensitivity to the place of religion in human life,” and though religion can feel official, it’s not official in the way that laws are. I also intuitively understand that religion involves mores: behaviors and opinions. C therefore looks correct, and better than A. But since this is such a difficult question, I would eliminate D just to be safe.
D definitely has to be wrong. De Tocqueville seems to think highly of mores, but is introduced in the passage as someone who was critical of an unrestrained entrepreneurial spirit. This is clearly stated in paragraph 5: “He appreciated the commercial and entrepreneurial spirit that Crèvecoeur had emphasized but saw it as having ambiguous and problematic implications for the future of American freedom.” Since de Tocqueville said that the entrepreneurial spirit threatened democracy, but mores helped sustain democracy, they can’t be the same thing.
The question stem nearly quotes paragraph 4: “For de Tocqueville, the optimism of the Enlightenment had been tempered by the experience of the French Revolution and its aftermath, and the prophesies of the early political economists were finding an alarmingly negative fulfillment in the industrial infernos of English mill towns.” The right answer has to be a part of this quote.I. the economic development of Americans. I is not explicitly mentioned in the quote from paragraph 4, so I can eliminate it.
II. the French Revolution and its aftermath. II is mentioned in the quote, so it’s correct.
III. conditions in English mill towns. III is also mentioned, so it is also correct.
A is wrong because American economic development is listed in paragraph 2 as a reason for Enlightenment scholars like Crèvecoeur to be optimistic: “Crèvecoeur wrote of the American that: ‘Here the rewards of his industry follow with equal steps the progress of his labour’ […] The rational, self-interested individual had emerged as Economic Being and, as such, was conceived as living most naturally in the conditions of a competitive market.” For economic labor to be called a “reward” suggests that it was a positive for Enlightenment thinkers, and so not something which should have given them caution.
B is also wrong because I is wrong.
C matches my prediction, so I can choose it and move on.
And D is wrong because I is wrong.
This is a challenging question to predict the right answer for, since it’s only telling me to compare Crèvecoeur and de Tocqueville, which is what the entire passage does. Since I don’t have any way to narrow my focus, I should just evaluate the answer choices.
This has to be incorrect, because while de Tocqueville expresses some concern about America’s future in the last two paragraphs, Crèvecoeur is nothing but positive about America.
B, however, seems to fit the passage. De Tocqueville expresses concerns, while Crèvecoeur only seems to have good things to say about America.
C is the reverse of B, and so is incorrect. It is de Tocqueville who is pessimistic, not Crèvecoeur.
D is wrong because it fails to take into account de Tocqueville’s concerns in the last paragraph.
Another broad question stem, so I have to just evaluate each choice and see which ones are and are not in line with the passage.I. Crèvecoeur believed that economic advancement was linked to individual self-interest. I is a direct echo of paragraph 2, where the author argues that Crèvecoeur saw that “The rational, self-interested individual had emerged as Economic Being.” So I has to be correct.
II. De Tocqueville believed that undermining American mores would threaten the country’s free institutions. II is just a summary of the last paragraph, so II has to be correct.
III. Crèvecoeur and de Tocqueville both predicted a swift demise for the American experiment. Since the answer choices don’t list an option for all three options to be correct, III has to be incorrect. And it is; Crèvecoeur does not suggest that anything bad will happen to America, and de Tocqueville doesn’t claim that America is at risk for a “swift demise.” So the answer must be I and II.
This is missing II.
This is missing I.
C has both I and II, so it’s correct.
And D has to be wrong because neither Crèvecoeur nor de Tocqueville believe that America is in for a “swift demise.”
This seems like a straightforward passage check question that directs me to the first paragraph, where the author states that the low road, “if the norm, must limit a nation’s economic competitiveness, living standard, and income equity.” It therefore feels like the right answer should be on that list.
The first paragraph does not mention anything on that list, so A has to be wrong.
The list is a list of aspects of a nation, so B makes sense as the answer. B also makes sense because the first paragraph starts to set up the problem that the passage is focused on arguing a solution for in paragraph 6: “For the United States to compete in an eventual global economy based on skilled workers and quality products, additional employer investment in training is needed now.”
Paragraph 2 and 5 state that technology helps economies to follow the high road approach, so it’s unlikely that these economies would be hurt by low road policies. That makes C wrong.
The passage implies that more developed nations are likely to have the resources to pursue the high road, but the passage does not focus enough on less-developed nations for D to be correct. So D must also be wrong.
Computers in the workplace are described in paragraph 5: “Workers who use computers on the job also earn more than do those of the same education level who do not use computers at work. Moreover, the earning difference increases with the level of technological competence.” So I’m looking for a weakener for at least one of these claims: either the claim that some computer workers earn less than workers who do not use a computer, or that one might not make more money as one gets better with computers.
There’s no comparison here between workers who use computers and workers who don’t, so A can’t be correct.
“More business” doesn’t necessarily equate to an earning difference; B allows for the possibility that the executives who correspond by letter could earn less than those who work with computers. Since B does not clearly weaken the passage argument, it has to be incorrect.
The arguments about computers focus on earnings, not on productivity, so I can safely eliminate C.
D, though, definitely does discuss earnings; specifically, it mentions that mechanics who do not use computers earn more than those that do. Since that contradicts the claim from paragraph 5 that people who use computers earn more than people who do not, D has to be correct.
This is really just a main idea question, since the author is using the passage to argue for a recommendation. Paragraph 6 clearly suggests what the author advocates: “For the United States to compete in an eventual global economy based on skilled workers and quality products, additional employer investment in training is needed now.” In other words, the author wants the US economy to use a high road response, rather than a low road one. That should be enough to find the right answer.
The passage is focused on employees and employers. The passage does mention government policies in paragraph 6, but even that needs to be focused on the employer-employee relationship, so I can safely eliminate A.
B is a great choice for the right answer since it matches the author’s recommendation in paragraph 6. I would choose it and move on.
Vocational education is mentioned in paragraph 5, but it is described as inferior to the training that individual companies offer: “Workers who receive formal company training command higher wages than do similar workers who attend only vocational school or receive informal on-the-job instruction.” That’s enough to tell me that the author would want less vocational education, not more, so C has to be wrong.
D is ultimately wrong for the same reason as C: because it doesn’t focus on the employer-employee relationship enough to resemble the passage author’s main idea.
The “muddy road” is mentioned in paragraph 2, when the author describes the kind of training employees receive: “About 40 percent of U.S. workers receive no formal training beyond a high-school education. They must submit to the contingencies of low-road employment, remaining at the periphery of the new economy. The remaining 40 percent of the workforce slog along the muddy middle road, getting some advanced education or job-related training but unlikely to enter the dynamic high-road labor market and attract employers.” So I’m looking for an answer choice which describes getting some advanced training, but still being unable to get a job which requires such training.
This sort of matches my prediction. A describes someone getting some “job-related training,” but who is also not likely to get a better job because they only qualify for one job. I’m not very sure about this, though, so I would eliminate the other answer choices before moving on.
This doesn’t resemble the quote from paragraph 2 at all, since it doesn’t really focus on a person’s ineligibility for jobs that require more training. So I can eliminate B.
This might be a way to get out of a muddy road situation, since getting more training, even if it is not formal training from their job, could help workers to enter the high-road labor market. But C does not describe an actual muddy road situation, so C has to be wrong.
This sounds more like a low road situation, since D doesn’t acknowledge the job training that helps characterize a person on a muddy road career path. That makes D incorrect, and A correct.
This reminds me of paragraph 4, when the author gives the example of quality control: “instead of checking the quality of the final product, high-road firms integrate quality standards in their automated production process, encouraging workers at all stages of their operation to demonstrate expertise and responsibility.” This is meant to be an example of how challenging the high road approach can be to take, so I’m looking for a situation that resembles this example from paragraph 4.
A would help workers to be responsible for quality control, which is almost a perfect match for the example from paragraph 4. It’s such a perfect fit that I’m a little suspicious of it; I’ll hold onto A, and just check to see if I can eliminate the other answer choices.
This doesn’t seem to resemble the example from passage 4 at all, since that example doesn’t talk at all about supervision. Instead, the example seems to be about moving away from something like B, so that individuals become more responsible for the products they create. That makes B incorrect.
C doesn’t immediately have anything to do with the production process, so I can eliminate C.
D has the same problem as C, so A has to be the right answer.
This is a pretty challenging question because the question stem introduces a potential weakener to the passage argument: that perhaps employers should not pursue high road policies because doing so will require resources that can’t go to employee salaries, thus making that employer’s workforce vulnerable to poaching from other companies who do not encourage such training. The question stem tells me I should focus on the author’s arguments about the high road, so I should keep the author’s main point in mind, as given in paragraph 6: “additional employer investment in training is needed now. Policies at all levels should encourage the coordination of employer-provided training and broader schooling.” The right answer will somehow echo this.
I’m not sure how A would help the employer described in the passage. The passage discussion of the high road focuses on the benefits of training, so talking about penalties for not training employees feels out of character for that key part of the author’s argument. And regulation doesn’t play a major enough role in the argument for me to be sure that the author would support it. I can therefore eliminate A.
B talks about training, which makes it attractive to me, since the high road is all about the value of training. And B would serve as a commonsense solution to the employer’s problem, since it could force employees to stay with companies that trained them until the company has benefited from that employee’s training.
This feels like the opposite of what the author would advocate. The author is totally for formal company training, while C seems to suggest that employers avoid training employees by simply poaching others. That’s enough to make C incorrect.
D doesn’t mention training at all, which is the central aspect of high road approaches, so D is also incorrect.
This is an implicit paradox question. It is phrased in such a way that I’m supposed to see the two assertions as in tension with each other. The right answer will help this apparent contradiction make sense. If the passage suggests that employers should invest in training, but training doesn’t create well-paying jobs, then one implication is that it will be possible for someone to receive training but not get a well-paying job. This is the author’s main concern in the last paragraph, which talks about how “a strategy of investment in human capital succeeds or flounders according to the availability of high-wage, high-skill jobs.” That should be enough for me to find the right answer.
The author does not say anything about how training would generate a large pool of skilled workers. The last paragraph also seems to imply that a large pool of skilled workers would be a bad thing: one that would threaten the viability of a high road approach. Since the author is unlikely to advocate such a threat to the author’s solution, A has to be incorrect.
B doesn’t seem to say anything about job availability, which the right answer needs to in order to explain the paradox in the question stem, so B is incorrect.
C has a similar problem to B, so I can eliminate it.
D, though, does mention the job market: specifically the availability of high-wage jobs. Since D matches my prediction, I can choose it and move on.
I remember that this statement is given in the first sentence, but on first glance, it isn’t explained. The explanation should immediately follow, but what does instead is a definition of what schemas are: “Schemas are organized knowledge structures in memory that can be thought of as generic concepts representing objects, persons, situations, events, sequences of events, actions, or sequences of actions.” It makes sense that the author would define such a central term to the passage, since people are unlikely to know it, but that first sentence also needs to be explained. That’s a really good hint that this is really a main idea question: that the passage will focus on arguing this point, which might take the length of the passage to frame and establish. Moving on to the next sentence, which is in paragraph 2, there’s a little bit more definition given—“A memory schema is activated when information similar to its content is processed by the cognitive system”—but that shifts into something that feels more pertinent to thinking and learning: “If the information is similar enough to the content of the schema, then it is judged an instance of what the schema represents. The schema is thus instantiated by the new information.” This brings to mind that the rest of the passage is about how schema help people to know how to act in situations that resemble situations they’ve been in in the past, such as in the restaurant example in paragraphs 3 and 4. That has to be enough for me to find the right answer.
Truth and falsehood are only lightly touched on in the last paragraph, which is probably why this choice is here: to tempt me with the last bit of the passage that I read before tackling the questions. But the possibility of erroneous memory is treated as almost tangential to the rest of the passage, which is focused on explaining how memory schemas affect our understanding of a situation based off of past experiences. Since truth and falsehood aren’t really discussed there, A has to be wrong.
This sounds like my prediction, and is one way of phrasing what the passage is focused on. I can choose B and move on.
C is definitely wrong because it’s actually referencing the same part of the argument as A is. Since both can’t be right, both have to be wrong.
The passage only discusses how memory functions in familiar situations, or at least situations that we’ve been in before. D is therefore out of scope, and has to be incorrect.
This is an assumption question, so I know I’m looking for something unstated in the passage, but that the passage argument referenced depends on. Assumptions can be tricky to predict though, simply because a given claim can utilize so many of them. So my best bet is to refer back to the specific argument about scripts, attempt to predict a correct answer, but to also be ready to go through the answer choices and ask whether or not the passage argument on scripts depends on the answer choice. Scripts are mentioned in paragraph 3: “There is a special type of schema called a script that not only aids in comprehending and remembering information but also helps guide behavior. For example, when a person is eating in a restaurant, the restaurant script activated in memory enables the person not only to expect certain events […] but also initiates behavior.” A clear assumption doesn’t jump out at me since this is almost more factual than argumentative, so I’m going to go through the answer choices.
Even though the beginning of the passage mentions learning, there isn’t really a lot of learning mentioned in the passage. More importantly, the argument about scripts I quoted above doesn’t clearly involve learning. It’s all about behavior, and knowing what to expect. Maybe that’s a kind of learning, but I can’t see how the argument about scripts involves anything about “new learning experiences,” since it is primarily about using previously gained information in a new, but similar, situation. That’s enough to make A incorrect.
There is no way that B is correct. The last paragraph mentions how schemas can include erroneous information: “this information is not retained perfectly.” And since scripts are a kind of schema, one has to assume that scripts are also vulnerable to this. I also can’t see how the argument about scripts would depend on perfect memory retention, since the script seems to operate in a more general way in paragraphs 3 and 4.
C is almost the exact opposite of what the passage says. A script is defined by its ability to be utilized in new instances: in visits to new restaurants, to use the passage’s example. Actually, that sounds like a pretty good assumption for the argument about scripts, since the idea that scripts are useful in new instances of a situation isn’t explicitly stated in the passage, but has to be at least part of the value of scripts. After all, if that weren’t the case, then why would scripts matter? That makes C wrong.
D matches the prediction I came to while thinking through C: that for the passage argument about scripts to make sense, scripts have to help people know what to expect in new instances of a situation. This is implied, but not stated, in the passage, which makes it an assumption. I can also test that this is an assumption by negating the answer choice; if D is an assumption, negating it should make the argument fall apart. So if I say that “schemas don’t help people to cope with new instances of a situation,” that makes the entire argument about schemas make a lot less sense. It’s not clear, for example, when a script would come into effect, or why. D has to be correct.
Given that this is about people having different memories of the same thing, I know I have to go to the last paragraph, which is the only one which discusses misremembering. Looking through that paragraph, nothing obviously stands out, in part because it’s mostly factual, and just about the fact that people can forget things. The only claim I can discern is: “Because many inferences are made during schema-based comprehension, some information may be recalled that was never presented.” That’s not really enough for me to make a prediction, so I need to instead test each answer choice and feel out whether or not the passage arguments about forgetting and misremembering offer support for the choice.
Inferences and schemas are both mentioned in the last paragraph, but I don’t see anything there arguing or even suggesting that the two are independent of each other. If anything, the effect of forgetting suggests the opposite. The fact that schemas do not perfectly retain information helps explain how erroneous inferences are made, which in turn suggests that inferences and schemas are related. A has to be wrong.
I don’t see anything about partial or impartial activation in the last paragraph, and I also can’t see what B has to do with forgetting or misremembering, so B is wrong.
I don’t see anything about the subconscious mentioned in the last paragraph, so C is out.
I wouldn’t have predicted D because it is such a mundane and definitional answer choice, but I can see how D has to be correct. The whole passage has been about how powerful schema are, and how well they draw on past information in order to help people navigate new instances of the same situation. It makes sense that the last paragraph would temper those claims just a little bit by discussing the fact that forgetting happens, since people intuitively know that they do not remember everything that has happened to them. To say that “some forgetting may occur” is to suggest that forgetting is normal, and not unexpected; that resembles D enough to make D correct.
Another broad inference question. All I can do is go through the choices with what I know about script activation from the passage: that it “enables the person not only to expect certain events […] but also initiates behavior.”
If anything, the passage seems to imply the opposite. To use the word “activated” when describing schema and scripts is to imply that they work like a switch when a similar situation is encountered. I certainly don’t see anything about how to will oneself to activate a script, so A has to be wrong.
B, however, fits the second half of the quote above more than well enough to make it correct.
Slot filling actually involves specific information: “information specific to the restaurant event being read is stored in memory as values for the particular actions, roles, and props involved in the restaurant script.” C is therefore incorrect.
“Depends” is a very strong word; arguing that A depends on B requires extensive and focused arguments unless that relationship of necessity is obvious. Inference are only mentioned in passing in the passage, which means that there isn’t enough focus on it for the passage to even imply an argument like D. That makes D incorrect.
I know that instantiation is mentioned in paragraph 4: “This process is called slot filling because information specific to the restaurant event being read is stored in memory as values for the particular actions, roles, and props involved in the restaurant script. The script is being instantiated.” I also get an implied definition in paragraph 2: “If the information is similar enough to the content of the schema, then it is judged an instance of what the schema represents. The schema is thus instantiated by the new information.” Since both describe instantiation occurring, the right answer has to be something that the two situations have in common. Both seem to involve new information interacting with a schema.
A has many of the components of what the right answer has to contain, but it comes apart at the very end. Schemas are described in the passage as preexisting in the memory. Since the passage discussion doesn’t really focus on new schema, A has to be incorrect.
The second paragraph implies that a schema is instantiated when new information resembles a schema, not when it alters it, so B has to be wrong.
The passage never talks about amounts of information; I have no idea how much information is needed to instantiate a schema. It’s too general for C to be correct.
D is the last answer choice left, and it works well. This is the condition in both the restaurant example and the description of schema activation in paragraph 2; instantiation should happen when new information resembles a schema enough to activate it. If the new information doesn’t fit the schema, the schema shouldn’t get activated. That means researchers can use D to determine if instantiation occurs.
My work on the previous question has helped me to focus on the notion that schema theory relies on past information to help learn how to deal with similar but new situations, so I’m probably looking for an answer choice that involves at least some of that. I also have an example of schema theory in action in the form of the restaurant example, so I’m also looking for a choice that fits what occurs there.
A is a pretty good candidate for the right answer. The restaurant example is all about “general information about restaurants” being used to help orient a person to a new situation. That sounds like background knowledge to me. But A is such a general answer choice that I would feel better about choosing it by eliminating the other answer choices.
Pronunciation—and more specifically, sound accuracy—isn’t really discussed in the passage, so B is easy to eliminate.
This is probably the most tempting answer choice here; perhaps even more tempting than the right answer. C mentions memory, and one can imagine feeling like schemas, which are repeated drawn upon, resembles rote memorization: memorizing something by repeating it over and over again. However, the passage does not discuss learning as occurring in this manner. One can imagine someone memorizing a specific script dictating how one will act in a restaurant, but without the natural mental processes mentioned in paragraph 4 occurring. More importantly, the passage describes a really rich interaction between past experience and new information that C simply lacks; we think of rote memorization as deadening and limiting, and being somewhat unforgiving of new information. That’s enough to make C incorrect.
Explanation is not mentioned in the passage in relation to schema activation. For D to be correct, I would need to know more about what those explanations involved. D could be right if those explanations included references to words and situations that the children were already familiar with, but it could also be wrong if those explanations were strict and clinical dictionary definitions. Since D does not give me enough information to make it a convincing right answer, I can eliminate it and confidently choose A.
This is a nice passage check question to begin a passage with. I know that regular people are mentioned in paragraph 2: “An essential element in this revelation is that the two primal beings each made a deliberate choice between good and evil, an act that prefigures the identical choice that all persons must make for themselves in this life.” So the answer must be something about having to choose between good and evil.
A matches my prediction so well that I can choose it and move on.
Ahura Mazda is one that is “existing eternally” (paragraph 1), but humans aren’t mentioned as also existing eternally, so B is incorrect.
Ahura Mazda is one of the primal spirits, so it doesn’t make sense that Ahura Mazda would have created itself. Ahura Mazda is also described in paragraph 1 as “the one uncreated god,” which also contradicts C.
This is incorrect both because it describes the Zurvanite heresy, and so is by definition not a belief held by Zoroastrianism, and also because Zurvan is described even in that heresy as “a remote first cause. He does not himself intervene in the present struggle between his ‘sons.’” So the passage even contradicts the other definition of subordinate as “following the orders of,” since Zurvan is giving no orders.
This is another quick answer choice to eliminate, even though this is an uncommon and sometimes difficult question type. To answer this question, I need to both determine what role the mention of Babylonian conceptions of cyclical time plays in the author’s argument, and then gauge what effect changing that claim would have on the passage argument. While the Babylonian roots of cyclical time are mentioned in paragraph 3, it is the idea of cyclical time itself, rather than who originated the idea, that the arguments in favor of the Zurvanite heresy draw on. The author’s argument about what made the Zurvanite heresy heretical could have stated that the Zurvanites themselves made up the notion of cyclical time, and the argument would still make sense. So the right answer has to mention that the new information would not affect the passage argument.
Since the question stem changes something in the passage argument, it’s hard to see how that change would generally confirm that argument. A is therefore wrong.
B is what I’m looking for, and so I can choose it and move on.
The passage argument does not depend on the notion of cyclical time coming from Babylon. All that matters is that the Zurvanites used the notion of cyclical time to form their heresy. That’s enough to make C wrong.
If C is wrong because the claim in the question stem is rhetorically immaterial to the passage argument, then D, which is a more intense form of C, will also be wrong.
The author’s argument is about what made the Zurvanite heresy a heresy. So it makes sense that a discovery that linked something foundational to Zoroastrianism to Zurvanite beliefs would weaken the author’s argument. In particular, since the author’s described rebuke of the heresy mentions that “declaring that Ahura Mazda and Angra Mainyu were brothers […] betrayed Zoroaster’s fundamental doctrine that good and evil are utterly separate and distinct by origin and nature” (last paragraph), I could imagine that a discovery which stated that they were siblings with Zurvan as their parent would also weaken the passage argument.
This is actually a part of official Zoroastrianism, so A would strengthen the author’s arguments about the religion, not weaken them.
A sounds like it would weaken the passage argument, which mentions that official Zoroastrianism held that “the concept of history” had an “end” (paragraph 3), but it’s important to note that the source is just a believer, and not someone who can justifiably speak for the whole religion. Since nothing in the author’s argument depends on every single follower holding the belief that history would end, B would not affect the passage argument, and so can be eliminated.
C fits my prediction. If such a text was found, then the Zurvanite heresy would not be a heresy, but rather something taught by Zoroaster himself. C is therefore correct.
D, though, would not weaken the passage argument. Whether or not Zoroaster knew the Zurvanite heresy was coming doesn’t clearly make it less likely that the Zurvanite heresy was incompatible with Zoroaster’s teachings. That makes D wrong.
This is a very general question stem, so I need to evaluate each answer choice with the main idea in mind: that the Zurvanite heresy was incompatible with Zoroaster’s teachings. However, the right answer could theoretically involve any part of the passage, so I need to be ready to check answer choices against the passage argument.
The passage is focused on Zoroastrianism, and does not therefore really mention other religions. A is too extreme to be correct.
B is an attractive answer choice because it is so broad; “some” is such a loose term that B would be correct if the passage argued that a single religion involved a wholly consistent doctrine. However, B has to be incorrect because consistency is only implicitly discussed in the passage, insofar as the Zurvanite heresy was inconsistent with Zoroastrianism. Even then, the passage does not imply that Zoroastrianism was “wholly consistent.” In other words, there is not enough information to make B correct.
Here’s another very broad answer choice that, like A, makes a statement about all religions. C has to be wrong for the same reasons A is wrong: because the passage is focused on Zoroastrianism, and is not making statements about religion in general.
D has to be correct and it is, since the Zurvanite heresy is an interpretation of a religious doctrine that is inconsistent with Zoroastrianism. Note that D uses “some,” that very weak word, which again only requires there to be a single instance of what it’s talking about to make the statement justifiable. Since the passage gives me one religious doctrine that allowed for inconsistent interpretations—Zoroastrianism—D has to be true.
Though this is a long and potentially confusing question stem, it’s actually just boils down to a pretty simple passage reference question. The situation in the question stem resembles the Zurvanite heresy, which means that the Orphic Greeks could have influenced the Zurvanites, or vice versa. I also have to be very careful because the right answer is asking about an impossibility—what cannot account for the Orphic Greeks holding this belief—while the wrong answer is asking about what could: not what is likely, or what must be true, but just what could possibly account for the Orphic Greeks believing in something similar to the Zurvanites.
This would account for the situation in the question stem; the belief above is similar enough to the Zurvanite heresy that the Greeks could have influenced the Zurvanites, or the Zurvanites could have influenced the Greeks. A is therefore incorrect.
While B is just the opposite of A, it ends up running into a similar problem; both describe a relationship that could explain similarities between Zurvanism and the Orphic Greeks. So B has to be wrong.
This is a pretty good candidate for the right answer on the face of it, since the Orphic Greeks believed in something that the Zoroastrians would have considered heretical. It is therefore unlikely that Zoroaster would have allowed the Orphic Greeks to influence them. But there’s also a really clear reason why C has to be correct: because the Orphic Greeks were around in the 6th century BC, or the 500s BC, while the first paragraph claims that Zoroaster was alive “between about 1400 and 1200 B.C.” Since Zoroaster predates the Orphic Greeks, there is no way that the Greeks could have influenced Zoroaster. C therefore has to be correct.
D is not an obvious candidate for the wrong answer, but there’s no obvious reason why D would be correct. Since the Zurvanite heresy came out of Zoroaster’s teachings, it is at least conceivable that the Orphic Greeks could have also developed similar beliefs to the Zurvanites if they were influenced by the same teachings. D is therefore possible, which makes D incorrect.
To answer this type of question, I need to gauge the rhetorical effect of the author’s response, and explain that effect. This part of the argument occurs late in the passage, and is a moment when the author stages a counterargument in order to dispense with it, and so strengthen the passage argument. The author’s response to the counterargument mentioned above is: “if I surrender my decision-making responsibility to anyone, that person could come up with anything, too. Since someone has to decide what is the right thing for me to do, it may as well be me. In fact, morally, it has to be.” In other words, the author points out that if letting individuals decide could lead to unpredictable moral decisions, that unpredictability doesn’t go away just because I decide to let someone else decide for me. The person I let decide for me—in this case, the previous paragraph and next paragraph implies, a politician or government official—will be prey to the exact same problems of moral judgment that I will be. So why should that person get to decide what I should do? That is a relevant objection; so relevant that the author can end with it.
A is immediately wrong because I know that the author’s response is relevant.
B has the same problem as A, so it’s also incorrect.
C matches my prediction. The person making the objection is implicitly assuming that politicians or government officials will be better at making decisions than the individual will: a claim the author strongly disagrees with. C is therefore correct, but since D also begins with “relevant,” I should double check and make sure it is incorrect.
While the first half of D is right, the second half is only a weak, if not tangential, implication of the author’s argument. The author is focused on what individuals should do; whether or not politicians obey their consciences, the passage argues, the individual still has to make moral decisions—such as judging whether or not an order given by a politician is morally justifiable or not—on their own. That’s enough to make D incorrect.
A quick glance at the answer choices tells me that this question will require me to assess the effect of new information on the passage: whether it strengthens a claim from the passage or weakens it, and how. This is an easier version of this question type, since it lacks an answer choice which states that the information in the question stem has no effect; that helps simplify my task and allows me to eliminate answer choices, since I can get rid of two just by figuring out if the information strengthens or weakens something in the argument. I know that the crux of the author’s argument rests on the idea that a person always has a choice: that even if I promise to do everything that someone tells me to do, I still have to make a decision to keep that promise every time that person tells me what to do. But if someone was incapable of breaking a promise, then they wouldn’t have a choice upon making that promise. That’s a pretty direct attack on the author’s argument, so I know it definitely won’t support the author’s argument, which means that I can eliminate A and B. Since I just have to eliminate one more answer choice, I would just move into the remaining answer choices now rather than spend more time trying to predict what the right answer will look like.
A has to be wrong because there’s no way that the new information would support the author’s assertion.
B has the same problem, so it’s incorrect.
This seems right, because that is the specific issue that the person described in the question stem faces: that they can surrender responsibility. My only concern with choosing C is that the person in the question stem still has to make a moral decision regarding who to promise to follow, and so is at least responsible for that moral decision, but the author’s point still depends on the idea that C captures. So while I will choose C, I will also quickly check D just to make sure it’s not correct.
I can safely eliminate D because the situation in the question stem changes something in the individual, rather than in the individual’s relationship with other people. Since “other people” aren’t mentioned in the question stem, D is wrong, and C is correct.
There’s not much strategy to this broad question stem on first glance. I just need to keep the main idea in mind, which was helpfully reinforced by the last question: that we are always responsible for our actions. But a quick glance at the answer choices shows me that each choice is about law, so the right answer will phrase the main idea in terms of the law. I can therefore at least predict that the right answer will have something to do with moral culpability: that every individual is responsible for their actions.
This choice is immediately attractive because it’s describing the primary scenario that the author considers: whether or not people can surrender their responsibility when the government tells them to do something. This is clear in paragraph 3—“You cannot hand over your autonomy willy-nilly to Fred or the government or anyone else”—and paragraph 5: “No government, no body of people, no position, no individual can have moral authority over any other individual.” The part that mentions according “with the will of the governed” is strange, but I can rephrase it in terms of the passage: that a law is only as good as the people’s willingness to follow them. I can imagine that the author would hold that people would only decide a law is legitimate if they believed it to be morally correct. Still, that second half of the answer choice worries me enough that I’ll eliminate the other answer choices just to be safe.
This goes directly against the author, who time and time again claims that people need to decide moral issues for themselves. I can therefore eliminate B.
C has to be wrong because it argues that the law is always good, and so one should surrender one’s ethical responsibilities to the government. This is effectively the same answer as B insofar as both advocate following the law rather than one’s moral compass when it comes to every individual decision that one makes, so C is also wrong. This is the case even though the author states something like C in paragraph 6; there, the author is simply stating as a potential fact that “in social situations order is beneficial” as part of a list of possible reasons why people would follow a law.
D is meant to be attractive because the passage does mention consulting experts in the last paragraph: “if you are ignorant, consult an expert.” But even there, the author drives home the point about individual responsibility: “but when to consult, whom to consult, and how to decide between conflicting experts is still the individual’s problem. And so is the final decision.” In other words, lawmakers still need to rely on their fallible judgment even when consulting experts; since there is no “either/or” proposition about consulting experts in the passage, while there is one in D—either you consult an expert or you rely on your fallible judgment—D does not accord with the passage, and so has to be incorrect.
Here’s another question where I get to eliminate two answer choices just by figuring out whether or not the new information strengthens or weakens a claim in the passage. The author’s argument depends on the idea that government officials are not better at making moral decisions than we are in order to argue that we are all responsible for our moral decisions. So if government officials are way better than normal people are at making moral decisions, that has to weaken the author’s argument; that means I can eliminate C and D. I can also predict that the situation in the question stem specifically weakens the argument that deals with government officials: “A government is made up of individuals who are fundamentally similar to me, and ‘to err is human’ applies to us all” (last paragraph).
The situation in the question stem would weaken the passage argument, so A can’t be correct.
B has the same problem, so I can eliminate it too.
This wasn’t what I predicted, but it does ring a bell. Specifically, C invokes the argument in paragraph 7: “if I surrender my decision-making responsibility to anyone, that person could come up with anything, too. Since someone has to decide what is the right thing to do, it may as well be me. In fact morally, it has to be.” But if I am surrendering my decision-making responsibility to someone who is demonstrably better than me at making moral decisions, that person is far less likely to “come up with anything” than I am. And so that at least introduces the possibility that people should not make moral decisions for themselves. I’ll still eliminate D just to be sure, but I’m strongly inclined to choose C.
The situation in the question stem doesn’t mention discomfort, so I can safely eliminate D and be confident in C.
This question caught me off guard: mainly because it sounds so much like the example of the traffic lights that the passage raises in paragraph 6. The only difference seems to be that the question stem adds the idea that some people drive on the right side of the road in other countries, but that’s not in the least bit different argumentatively from the claim about a law stating that everyone drives on the left. It’s so puzzling that it makes sense to use the answer choices to help me tackle this question, and luckily the answer choices tell me that I’m being asked about the “central thesis of the passage.” Even if I take the claim in the question stem to be an added piece of evidence in support of the claim made in paragraph 6, the argument in that paragraph is not central. The central argument is that “responsibility for your actions is not transferable” (paragraph 2), while paragraph 6 argues that “the responsible individual will often have to take into account what other individuals are doing when that individual is calculating the consequences of her or his action” (paragraph 6). That’s a related, but ultimately different point, which tells me that the right answer is going to be one which claims that the information in the question stem does not affect the central thesis.
At worst, the new information in the question stem has no effect, and at best it supports a subsidiary or tangential argument in the passage. It does not directly affect the main idea of the passage, so I can eliminate A.
B matches my prediction, and so I can choose it and move on.
C has to be wrong: both because the situation above only tangentially relates to the main idea of the passage, and also just because there’s no easy way to read the information in the question stem as damaging the main idea, since it doesn’t obviously attack the notion that people should be responsible for their own ethical decisions.
D is a weaker version of C, but is wrong for the same reason: because the information in the question stem is, in no obvious way, damaging to the passage.
This is a broad assumption question, which means that it is difficult to predict what the right answer will look like. So my best strategy here is to make sure I know the main idea regarding male and female experience—stated in paragraph 1, that “such experiences differ, there can be no disagreement; but that such experiences overlap, there should also be no disagreement”—and be ready to test each answer choice: to determine if the answer choice is unstated, and if it is necessary for the passage argument to make sense.
The quote from paragraph 1 directly contradicts, A, so I can eliminate it.
B almost sounds like the quote above, but there’s enough meaningful difference between saying that the male and female experiences “overlap” and what B says. I can also see that B has to be assumed in order for the passage argument to make sense. I can test whether or not an answer choice is the right answer to an assumption question by negating the answer choice; if I add the negation to the passage argument, the passage argument should no longer be supportable if this is the right answer. This is because an assumption has to be a necessary claim made by the passage. If I were to say “there is no degree of similarity between female and male experience,” that would directly contradict the quote from paragraph 1. That’s therefore enough to tell me that B has to be right answer.
Given that the passage argument here is working to define the female experience and female fiction, there is absolutely no way that C could be correct.
The passage does not mention anything about how the female experience influences the male experience. The passage does, however, imply the opposite in the description of Wide Sargasso Sea. In paragraph 4, the author describes how the female experience is affected by male experience: in the reduction of the woman into “an object, living out others’ sense of her experience, not her own.” That makes D incorrect.
This is a quick passage check question that refers me to paragraph 2: “There is by now a sizable body of fiction that focuses on female experiences or conditions, in which women must find their way personally, professionally, socially, in what is basically a patriarchy.” That should be enough to find the right answer.
A matches the quote from paragraph 2, so I can choose it and move on.
Style is barely mentioned in the passage, which already makes me suspicious of B, especially given how strong of an answer choice A is. But B is definitely wrong by virtue of the quote from Showalter, who says that one should not assume that female writers “display stylistic resemblances distinctively feminine.”
The passage does not include any female characters seen from a man’s perspective, so I can eliminate C.
The passage gives just one example of a female character who serves as an emblem of female experience: Bertha in paragraph 4. But the passage doesn’t talk about many novels doing this, so without a more direct statement in support of D, I have to eliminate it.
Another nice basic comprehension question. I know Jean Rhys is brought up in paragraph 4 as an example of a writer who is doing something that the passage author respects through her depiction of Bertha: “There is, I feel, no male novelist who could have picked up the thread of Bertha’s existence and turned it into an emblem, as Jean Rhys did; and here alone we note the way the female novelist can perceive aspects of experience that remain (at least in our era) outside the reach of the male writer.” I’m especially paying attention to that quote because it’s one that directly states the author’s opinion, since the author is willing to say “I feel” when making that claim. That should be enough to find the right answer.
A almost perfectly matches the quote from paragraph 4, so I can choose it and move on.
There isn’t a clear sense that Rhys is making a new literary form: that is, an entirely new approach to literature that other people will take on. The author’s point is more focused on Rhys and one book, so B is too broad to be correct.
Answer choices like C are meant to be tempting because they echo the passage, but also distort it. The quote from paragraph 4 talks about the female experience, and not a prose style, so C has to be incorrect.
The quote from the author in paragraph 4 strongly praises Rhys, and the argument in paragraph 4 suggests that Rhys is doing something good in with Bertha’s character from Jane Eyre. Since D is such a negative answer choice, it can’t be correct.
It’s funny how so many questions can sometimes draw on the same piece of the passage. In this case, though, I need to be ready for the options to reference both paragraph 3 and 4: both Jane Eyre and Wide Sargasso Sea.I. could have been explored equally well by male or female novelists. My work with the last question helps me rule out I automatically, since the author says that “no male novelist” could have done what Rhys did with Bertha (paragraph 4).
II. provided Jean Rhys with an archetypical symbol of the plight of women. That quote from paragraph 4 also helps me to choose II.
III. functioned for Charlotte Brontë primarily as a plot device. III, though, seems to contradict II, but that’s not a reason to eliminate it. After all, the author brings up two tonally opposed usages of Bertha in Jane Eyre and in Wide Sargasso Sea. In paragraph 3, the author states that Bertha “is presented as the element that must be eliminated in order for Rochester and Jane to complete their destiny together. Imprisoned in the upper reaches of Thornfield, she is a threat to foreground order and stability, a principle of chaos, in fact.” In other words, she is a plot device for Brontë. That’s enough for me to know that I’m looking for a choice that mentions both II and III, but not I.
I is definitely wrong because of what the author says in paragraph 4.
B mentions I, so it’s wrong.
Same with C.
D matches my prediction and does not mention I, so I can choose it and move on.
Virginia Woolf is mentioned as an authority in the last paragraph, so I definitely took notes on what she had to say during my initial reading of the passage. In that, she mentions that “women had to develop a prose of their own. A quick scan of the paragraph for evidence in favor of the argument mentioned in the question stem brings me to the end of the paragraph: “There is no reason to think that the form of the epic or of the poetic play suits a woman any more than the sentence suits her. But all the older forms of literature were hardened and set by the time she became a writer. The novel alone was young enough to be soft in her hands.” In other words, it is not that women are less well-suited to epics or poetic plays, but rather that the novel is a young enough art form for women to change it to better suit the female experience.
A matches the end of the quote from the last paragraph well enough to make it correct. To say that the novel “was young enough to be soft in her hands” is to say that it is recent (young) and flexible (soft in her hands).
This is incorrect, mainly because while Woolf brings up male writers and their sentences, that reference is not directly connected to the novel enough for B to be correct.
Woolf says the opposite: that the novel was the newer genre. C has to be wrong.
D is actually wrong for the same reasons B is wrong. Both B and D are a reference to a related, but different argument that Woolf makes about the need for women to make prose that is their own. Those arguments are ultimately independent from the author’s claim that the novel is well-suited for women writers because the novel form is so young.
Here’s a straightforward main idea question. The author begins by talking about similarities between the state, religion, and sport, but in paragraph 3 uses an “I” statement to highlight the passage’s main point: “I am saying that sports flow outward into action from a deep natural impulse that is radically religious: an impulse of freedom, respect for ritual limits, a zest for symbolic meaning, and a longing for perfection.” The only problem is that that’s not a recommendation, which is what the “should” in the question stem implies the right answer needs to be. Looking through the passage, though there’s a lot about religion and sport throughout, the author pulls back a little bit on idea that sports are a religion in the last paragraph: “I don’t mean that participation in sports, as an athlete or fan, makes one a believer in ‘God.’” Instead, the author seems to be saying that sports aren’t just play: “In the language of Paul Tillich, sports are manifestations of concern, of will and intellect and passion. In fidelity to that concern, one submits oneself to great bodily dangers, even to the danger of death.” So I’m looking for an answer choice which argues that we should see sports as being pretty important.
A matches my prediction, and so it’s correct.
The passage mainly talks about the history of sport and the state and religion in the first paragraph, but it quickly moves on to talking about religion and sport more generally. This is done in order to argue that sports are important insofar as they satisfy many of the same needs that religions do. Since the passage argument moves on from B, B can’t be the “primary” message of the passage, and so is incorrect.
C is wrong for the same reasons that B is wrong, but even more so. The patriotic aspects of sport are limited to just the first paragraph, and do not play a role in the rest of the passage.
The author actually suggests that one need not make this distinction: that instead, sports draws on the blurry boundaries between ceremony and sports. That makes D wrong.
This is a quote from the last paragraph, but I don’t really need more than the general passage idea to answer this question, especially because of the work I did to answer the previous question. I know the author does not use the term to specifically argue that sports are a religion because the quote takes place in the last paragraph, where the author starts to move away from that claim. Instead, the author is making a more “generic” point about sport: one that says sports are not meant to be restrictive in the way that religion can be.
A is too religious in tone to be correct. The author is talking about sport here and distinguishing it from religion, so I can safely eliminate A.
B, though, sounds like a good candidate for the right answer. I know that the author is interested in distinguishing sport and religion in the last paragraph in part to make the claim more “generic” (last paragraph). Still, B is not exactly what I was expecting, and so to feel better about choosing B, I would eliminate the other answer choices.
C has to be wrong because I know that the author is moving away from this idea in the last paragraph.
D is actually a pretty similar answer to C, insofar as both play up the connection between sports and specific religions, when the author is trying to downplay that connection in favor of a more general claim. That makes D wrong, and B correct.
Though this question resembles the previous one, it uses a far more obscure reference: one not directly attributed to a specific speaker, and so one that I am likely to have only skimmed during my initial reading. The question phrasing also suggests that this is asking me more about the definition of the phrase, rather than just the argument of the paragraph, so I should reread the part of the passage where the quote is from. The quote occurs in the first paragraph: “The ancient Olympic games used to be both festivals in honor of the gods and festivals in honor of the state—and that has been the classical position of sports ever since. The ceremonies of sports overlap those of the state on one side and those of [religion] on the other.” The author here is making a point about history: that sports retain something of the character they exhibited since the ancient Olympic games. That should be enough for me to find a right answer.
The reference in the question stem is to sports, not athletes, so I can eliminate A.
B has the same problem as A; it is talking about athletes, not sports.
C matches my prediction, since it mentions the way that the history of sports seems to help explain how it works today. I would choose it and move on.
D would be a tempting answer choice if C wasn’t so clearly right, and if D didn’t use some words that are difficult to align with the specific quote the author is asking about. All I know from the quote is that “The ceremonies of sports overlap those of the state on one side and those of [religion] on the other.” I don’t know if athletics is “dominated” by religion and politics today, or even back then. “Dominate” is such a provocative word that I would need a specific argument explaining what that means in terms of sport, and specifically mentioning this overwhelming influence. Since the passage lacks that, or any explanation for what that has to do with the “classical position” of sports, D is worse than C.
This is a straightforward weakener, so the right answer has to somehow attack the notion crystallized in the last paragraph: that sports addresses many of the needs that religion also fulfills. So I should expect the right answer to say something about how sports doesn’t fulfill religious needs.
This might sound intuitively correct, but the passage never argues that sports are more popular than religion. It is instead making an argument about how sports functions, and what it does. Since that doesn’t obviously depend on people liking sports more than religion, A does not weaken the passage arguments.
The way the passage argues—the way it treats the link between sport and religion as if it is a revelation—seems to imply that most sports fans aren’t aware of the links between sport and religion. So B wouldn’t weaken the passage argument, since it seems at worst in line with it.
The “only” in C might make C feel extreme, but extreme answer choices are pretty common in weakener questions. Making absolute claims that contradict the passage argument is a classic way to weaken an argument. In this case, I can tell C weakens the argument because it matches my prediction. If only religious people experience the spiritual dimension of sports, that implies that sports doesn’t fulfill people’s religious needs more broadly. It increases the likelihood that the religious dimension in sports is only there for those who are predisposed to see the world in a religious way: that sports doesn’t actually function in a similar way to religion, but is just a mirror of it. That’s enough to make C the right answer.
I know D is wrong immediately because the passage barely ever talks about athletes, and does not clearly distinguish between athletes and fans. Nothing in the passage depends on athletes and fans feeling as enthusiastic about sport as each other, so I can safely eliminate D.
This is the last sentence of the passage, but it is given as an extrapolation of the previous sentence: “In fidelity to that concern, one submits oneself to great bodily dangers, even to the danger of death. Symbolically, too, to lose is a kind of death.” I know that the author is not talking about actual death here, since the author says this is a symbolic death. But the previous sentence also makes me aware that the author is talking about the intensity with which one approaches sport: that one goes into it being willing to be hurt by it. So I’m looking for an answer choice which mentions intensity and dedication.
A fits, since I would not “submit” myself to “great bodily dangers, even to the danger of death” in sports if I was not emotionally invested in sports. I would choose A and move on.
While the quote mentions loss, the last paragraph does not mention honoring anyone, so B has to be incorrect.
C is definitely one way one can read the quote, but it’s not a way that the last paragraph supports. Nothing in the last paragraph mentions that it is unacceptable to lose in sport; if anything, the passage arguments about how we are willing to suffer in sports implies that the agony of loss is an accepted part of competition. C is therefore incorrect.
There is no evidence that the passage considers the last sentence to be a statement of exaggeration. It is a statement of intensity; for it to be exaggeration, the author would have to imply that there was something wrong with that approach. Since the author actually seems to embrace that last statement, D has to be incorrect.
This claim is made in paragraph 2, which my initial reading tells me is all about how selfish people are incapable of “friendship of the most morally excellent kind.” Specifically, the quote from paragraph 2 is: “His giving could be a minor concession for her serving him or even a further expression or assertion of his power over her and of her dependence on him.” In other words, giving could be an assertion of power when it reinforces a relationship of subservience. When a boss gives a worker their pay, that can be read as an assertion of power: that money that the worker needs will only come from the person above them. A quick glance at the answer choices tells me that I will need to come up with an analogous situation to the one in the question stem, so I’m looking for a situation in which someone gives something in a way that reinforces that the giver is above the receiver.
A works. It implies that the judge would not give lighter sentences to criminals who do not show respect for the judge: who do not respect the judge’s power over them. A is also in line with the argument of paragraph 2 insofar as it is a fairly selfish reason for a judge to lengthen or shorten a criminal’s sentence.
B seems like a tempting answer choice because there is a clear power structure here, insofar as the boss is using power over the employees to get them to do something. However, the major difference between B and the passage is that the question is asking about the part of the passage where the husband gives something to his wife, and it’s unclear what the boss is giving the workers. That’s enough of a difference from the passage to make B incorrect.
C is selfless, while the man referenced in the question stem is selfish, so there’s no way that C is analogous. C has to be incorrect.
D differs from the part of the passage referenced in the question stem because it’s not a clear example of someone giving in order to reinforce their power over another person. D sounds way more collaborative than the situation in the passage, so it has to be incorrect.
The question stem tells me that the right answer will be stated without evidence or support. That means that the most efficient way I can approach this question is to check each answer choice and eliminate answer choices which the passage gives evidence or support for.
This claim is given in the middle of paragraph 1. It is followed by an explanation for why this is the case: “If someone were genuinely able to care for another person for the other’s own sake […] then he or she would not be selfish.” This elaboration is a form of support, so A has to be wrong.
This claim is made in paragraph 3, and is elaborated on with three examples: “there are important aspects of friendship besides caring for the other—i.e., enjoying being with the other person, or sharing certain kinds of activities, or liking the other person.” Each of those examples helps to prove that “There is more to friendship than caring for the other,” so B has to be incorrect.
C is meant to be tricky, since “beneficent” comes up at the end of paragraph 2. Since nothing follows C, one might guess that C is unsupported. However, C is just the summation of the argument made in paragraph 2; it is explained by the example of the selfish man who still does a lot for his wife. That makes C wrong.
D has to be right since it is the last choice standing, and it is correct. This claim comes at the end of paragraph 3. Since it is the end of a paragraph, no support or explanation can come after it, and since it is prefaced by “Second,” I know it is its own discrete point that starts in that sentence, so no evidence for the claim can come before it. The second half of the sentence—“be well disposed toward others”—is not evidence for it, or explanation: it is just a restatement in different words. It is not, in and of itself, an argument or evidence in favor of D. Since D is without support, D has to be correct.
The example of Dave in the last paragraph is meant to help argue for the claim at the paragraph’s end: “This is why the caring and the acts of beneficence are not separate from my own interests, from what is personally a good to me. In fact, friendship is a context in which the division between self-interest and other-interest is often not applicable. In other words, the claim is meant to suggest that when it comes to friendship, what is good for me and what is good for my friend might not make sense to distinguish. So I should be prepared for a weakener to that claim: one that says, perhaps, that there are reasons to distinguish what is good for me from what is good for my friend.
This is a point made in the previous paragraph, but it does not clearly have a lot to do with the author’s argument about how it might not make sense to distinguish between self-interest and other-interest when it comes to friendship. Since the situation in the question stem does not relate enough to A to weaken it, I can safely eliminate A.
B matches my prediction well. If Dave and the author argue about how they should spend time together, then that’s a clash between self-interest and other-interest. Their arguments make that situation from the question stem applicable enough to B to make B correct, since it provides a situation in which that line might matter.
C doesn’t really have a lot to do with the reference to Dave in the last paragraph, so it’s not clear how C would weaken that. However, I can see how the situation in the question stem would strengthen C, since it could be read as an example of two people who are not friends in the fullest sense. I can therefore eliminate C.
Just because Dave and the author argue, that does not suggest that the author’s friendship with Dave would not define what is important to the author. If Dave and the author are arguing, and they are friends, one can assume that the argument could involve something important to them both. Since I can’t clearly establish how the situation in the question stem would weaken D, and I can see how it might strengthen D, I should eliminate D and choose B.
This refers to the same part of the passage as the last question did. However, this question is asking me about entailments of the last paragraph: that is, claims that I could make that justifiably build on the arguments made there. Since one can make a myriad of arguments on the basis of any individual claim, my best strategy here is to just go through the answer choices and figure out if they are related enough to the claim mentioned in the question stem to be correct.
A seems like a pretty good candidate for the right answer. If friendship helps equate self-interest and the interests of one’s friends, then it means that friends tend to have the same interests. Having the same interests, logically, should “foster cooperation” and “reduce antagonism,” since the friends are likely to lack the opposing needs needed to have antagonism. That’s enough for me to choose A and move on.
The passage talks about people who are incapable of being friends in the fullest sense, but I don’t remember anything in the passage about how one should act with people who one is not friends with. That’s enough dissimilarity from the passage to make B incorrect.
C is a statement about history, but the part of the argument mentioned in the question stem doesn’t seem to necessarily involve history. It is very possible to share interests with someone that you just met, which is one way to blur the distinction between self-interest and interest in one’s friend. C is therefore incorrect.
D is actually a very similar answer to C, insofar as both involve elements of time and history that the passage argument mentioned in the question stem does not. Since C and D are rhetorically similar, I can eliminate both.
This is a broad question stem, but the answer choices tell me that I’m meant to apply what the passage argues about “the reciprocity and transitivity of friendship” to actual people. The question stem is again referring to the last paragraph, which also gives me an example of actual people and friendship: “In caring about the weal and woe of my friend Dave, it is integral to the nature of this caring that it be for someone whom I like, whom I know likes me, who cares about my weal and woe, whom I trust, who is personally important to me, who cares about our friendship, etc.” In other words, what’s important in a friendship is that I care about someone who also cares about me in the same ways. One answer choice should mention this.
A is a reference to the “transitivity” of friendship, but nothing in the passage discusses how one can be friends with a friend of your friend. Everything in the passage is about how one person can or cannot be fully friends with another; since nothing like A is discussed in the passage, A has to be incorrect.
This is a really tempting answer because it seems, on the face of it, to involve the reciprocity that the author describes in the last paragraph. However, it’s worth noting that nothing in that list says that friends must satisfy the same needs in each other. That means while there is some reason to choose B, I’m very unsure of it, so I would hold onto B, but be ready to eliminate it once a better answer choice comes along.
C instantly makes me suspicious, because it could be implying that Mary is selfish, and I know from paragraph 1 that “A truly selfish person could not have friends in the fullest sense.” However, I don’t necessarily know that Mary is selfish; it might just be that Bill likes to give, and Mary likes to receive. But C has to be incorrect because it doesn’t really speak to reciprocity or transitivity in friendship. I don’t have any examples of such unequal friendships in the passage, while I do have a last paragraph which talks about the importance of people giving as much as they are receiving, so C is a worse answer choice than B.
D, however, is a better answer choice than B because D is actually a weaker claim than B, since B involves Mary and Bill satisfying the exact “same needs” in one another, while D just requires that the two be friends, whatever that means. That means that D better resembles the example of Dave and the author in the last paragraph, so I would choose D over B and move on.
The first paragraph argues that art was seen as synonymous with the needs of the ruling classes who paid for art because artists were trained to understand art in that way. The author builds on this point in the second paragraph to argue that it was difficult to make art that expressed “the experience of other classes.” This argument about how difficult it is to do this is continued in the next paragraphs, where even the dismantling of the artistic tradition which trained people to see art as what rich people thought of as art only produced art that reflected a narrow range of experience. Since the one consistent claim argued throughout the passage is that artistic training is to blame for the narrow range of experiences that art captures, that has to be the main argument of the passage.
This is only argued in the first two paragraphs. Paragraph 3 mentions that this interrelationship changes in the 20th century, but the problem of democratizing the experience of art remained. Since A only covers a part of the passage, it cannot be the passage’s main argument.
B is also mainly discussed in paragraph 1, and so is as limited as A is. B is therefore also incorrect.
C matches my prediction, and runs all the way through to the passage’s arguments about abstract art. I can therefore choose C with confidence and move on.
This is barely implied by the second half of the passage, and does not discuss the effect of artistic training, which is definitely a key aspect of the passage argument. D has to be incorrect, then, because it does not describe a part of the argument that would be considered the main part of the argument.
The most prominent reference to “tradition” is in the start of the fourth paragraph: “One of the reasons the potential freedom gained by the dismantling of the tradition was not used may be the way painters were still trained.” But that’s in part a reference to paragraph 3—“The crisis provoked by those who tried to extend the area of experience into which painting might be open continued into the 20th century. But its terms were reversed. The tradition was indeed dismantled”—so what was dismantled? Looking back to the first half of the passage, it has to be the tradition of artists becoming skilled in using conventions that corresponded closely with “the class the artist was serving.” So that’s probably what the right answer will say.
The author mentions eternal and artistic truths as a reference to how artistic training narrowed peoples’ understandings of what art could be. But “tradition” is a term of history; it’s a claim that something stayed in effect over time. And it’s not clear that this belief that art was about recording and preserving “eternal artistic truths” was either maintained or discontinued. In other words, the passage does not clearly refer to A enough for A to be correct.
B, though, matches my prediction, since it describes the tradition mentioned in paragraph 1 that gets dismantled in paragraph 3. I would choose B and move on.
This is mentioned in paragraph 2, but it is the author’s specific criticism of Ford Madox Brown’s Work. The author isn’t clearly suggesting that there is some tradition of depicting subjects in this way; the author instead argues that the mythological or symbolic are just some of the ways that the conventions held by wealthy people concerning what art was limited artists’ ability to depict the experiences of others. Since there’s not enough specific information regarding C and artistic tradition, C has to be wrong.
D is very clearly mentioned as a new development in art, and one that takes place after “the tradition” was dismantled (paragraph 4). That means there’s no way that this expansion could be considered part of the tradition, which makes D incorrect.
I know from my initial reading of the passage that the author is concerned that artistic teaching is too narrow: that conventions limit the ability of art to portray a wide range of experiences. So I know, then, that the author would most want art to either portray a wide range of experiences, or to defy conventions.
A matches one of my predictions, so I can safely choose it and move on.
This is mentioned in paragraph 1, and is associated with the period when art was strongly tied to the perspective of the wealthy. Since the author seems to fully reject that period and what art was then, I can safely eliminate B.
C is also from that first paragraph, so I can eliminate it for the same reasons I eliminated B.
The author claims that the religious thematizing of the working class was a failure of Ford Madox Brown’s Work, so there’s no clear reason why the author would think D was a good thing.
This is a quick passage check question that sends me to paragraph 1. The author mentions how artist training is to blame: “the artist became skilled in using a set of conventions […] And these conventions corresponded so closely to the social experience-or anyway to the social manners-of the class the artist was serving, that they were not even seen as conventions but were thought of as the only way of recording and preserving eternal truths.” The right answer needs to echo something from this quote.
A would imply that artists were somehow studying the ruling class, but that is not clear from the first paragraph at all. I can therefore eliminate A.
This is an attractive answer choice because it almost echoes the end of the quote from paragraph 1. But the quote talks about methods, and not about experiences. The passage does not clearly suggest how the artists felt about the ruling class or their experiences, so B has to be wrong.
C also isn’t mentioned in that first paragraph. There is no clear sense that artists could even grow wealthy as artists, or that emulating the wealthy was some way to become wealthy too. So C has to be wrong.
D is the last answer choice left, so it has to be right, but I don’t think I would have chosen D if it wasn’t the last answer choice. The passage doesn’t actually explicitly mention this motivation; it states that artists served the wealthy, but that’s more of a statement of fact than it is an explanation for why artistic convention resembled the conventions of the wealthy. It is very possible to serve someone or be supported by someone and to not take on their mannerisms. But, again, D is the last choice standing, and A-C are very clearly incorrect, so I’ll just have to accept that this is the right answer.
The first sentence of paragraph 2, which is where Brown’s painting is mentioned, gives me the answer to this question: “During the 19th century certain artists, for consciously social or political reasons, tried to extend the professional tradition of painting, so that it might express the experience of other classes. Their personal struggles, their failures, and the opposition they met with, were a measure of the ambition of the undertaking.” So I know Brown is meant to serve as an example of one artist who tried to depict the experiences of classes that were not wealthy. But the middle of the paragraph goes on to say that Work “will give some idea of the extent of the difficulties involved.” So I’m looking for an answer that mentions how the attempts to depict the experiences of other classes failed, or was difficult to do.
A seems like a good match for my prediction. Brown tried to expand what professional art depicted, and found it difficult to do. I would choose A and move on.
While the next paragraph shifts to talking about abstract art, the passage offers no comparison regarding what forms of art are more “powerful,” so I can safely eliminate B.
Brown’s example is very specifically mentioned as being limited to the 19th century, and so doesn’t tell me about how things are today. My main problem with this question is that C sounds a lot like A, since A doesn’t claim that its findings are limited to the 19th century, but C makes a far more specific claim than A does. When a choice is phrased as C is—that is, when it specifically says that a problem is “still with us today”—the answer will only be correct if the passage specifically states that that is the case. Since the passage does not, C is worse than A.
While the ruling classes are present in paragraph 2 by implication—in the form of the challenges that Brown faced when trying to paint working class subjects—the example doesn’t involve the dynamics between Brown and the working class, so D can’t be correct.
The passage section on type-type identity theory is much more difficult to comprehend than is the author’s discussion of behaviorism, so it makes sense to start with the latter in order to determine what concern they have in common. The author’s criticism of behaviorism occurs in paragraphs 2 and 6. In paragraph 2, the author suggests that mental phenomena can occur that are against our dispositions: “You may have a pain in your toe without having a disposition to complain or move your toe because you may be a super Spartan who does not want to give any sign of what you are feeling,” The author then raises the claim that “behaviorism takes too external a view of the mental […] mental states and processes are not behavioral dispositions; they are the internal states and processes that are responsible for relevant behavioral disposition.” This gets echoed in the last paragraph, where the author complains that behaviorism oversimplifies how reactions work: A particular belief taken by itself cannot be identified with particular behavioral dispositions because the relevant behavioral dispositions are determined by various beliefs taken together plus various desires and other mental states.” As complex as that sounds, the author gives the example of the umbrella to clarify: how your decision to take an umbrella will obviously depend on your sense that it might rain, but it will also involve your willingness to get wet, how far you’re going, and various other beliefs. Type-type identity theory is first defined in paragraph 4: that theory “holds that every type of mental [experience], such as pain, is identical to a corresponding type of physical [event], such as the firing of C-fibers. The author’s somewhat reserved criticism for this theory immediately follows: “It is controversial whether this is an overly strong thesis because of its implication that the physical basis for any type of mental [experience] must be the same in different creatures.” In other words, the author is concerned here that type-type identity theory requires there to be a really strong relationship between a single specific physical event and a mental phenomenon. The common concern in the passage between behaviorism and type-type identity theory, then, is that both reduce mental phenomena down to single causes. That should be enough for me to find an answer.
That’s the case with type-type identity theory, but behaviorism is more about mental tendencies. A quick scan of the passage confirms that behaviorism is the opposite of A: reducing physiological states—like having a stubbed toe—into mental states, or habits of thought.
Both theories provide reasons for why pain exists, so B has to be wrong.
C is a pretty good candidate, because it is saying that both need to consider more than one cause for a mental state. Behaviorism ignores other beliefs and feelings in favor of a single tendency or behavior, while type-type identity theory ignores the possibility that the same mental state can have multiple and different physical events, as well as beliefs. Though this is a difficult question, C helps me to better understand the passage, and aligns with my prediction, so I can choose it and move on.
This is the opposite of A, but amounts to the same problem. This only describes behaviorism, and not type-type identity theory. It therefore has to be incorrect.
The question stem describes a situation in which a chemical is shared between humans and earthworms. That chemical implies that earthworms can feel pain, since that chemical relieves pain in humans. My work on the last question helps me to see how this resembles type-type identity theory, and very specifically the author’s concern over it in paragraph 4: “It is controversial whether this [type-type identity theory] is an overly strong thesis because of its implication that the physical basis for any type of [mental] experience must be the same in different creatures.” Since the question stem is describing a person who believes the presence of the same physical basis (opioid peptides) in different creatures causes the same mental experience in those creatures (pain), the right answer has to be that the person is a type-type identity theorist.
A is wrong because it doesn’t match my prediction. But it’s also wrong because nothing in the question stem mentions or even resembles the passage discussion of behavioral dispositions or tendencies, which are key to behaviorism.
B matches my prediction, so it is correct.
A functionalist is described in paragraph 4 as a sort of less extreme version of the type-type identity theorist, but functionalist would claim that “it does not matter what mental states and processes are made of, any more than it matters what a carburetor or heart or chess king is made of.” Instead, functionalists would look at pain as related to “a whole system of interdependent functions” (paragraph 5). A functionalist would likely therefore not jump to the conclusion that the presence of a single chemical is enough to suggest that earthworms feel pain.
The passage doesn’t discuss nonscientists, so D is easy to eliminate.
The fact that the question doesn’t just say “behaviorism” here, but instead says “crude behaviorism” turns what could be a really broad question into one that I can predict the answer to. To include “crude” is to suggest that the question is asking me about what makes behaviorism crude, which is another way of saying flawed. Given my work on the first question in this set, I know that the behaviorist flaw is crystallized in the last paragraph: “A particular belief taken by itself cannot be identified with particular behavioral dispositions because the relevant behavioral dispositions are determined by various beliefs taken together plus various desires and other mental states.” In other words, behaviorism fails to account for how other feelings and thoughts also affect our dispositions: that we should not reduce behaviors down to single causes.
A fits that description, since taking internal context into account would mean taking “desires and other mental states” into account. A is therefore correct.
The author also points out a flaw in type-type identity theory in order to instead advocate for functionalism, so B has to be incorrect.
The passage doesn’t distinguish between predictable and unpredictable actions, so C can’t be right.
And D goes too far. Behaviorism doesn’t explicitly require one to hold that beliefs and feelings don’t exist. Behaviorism simply ignores them by instead focusing entirely on individual “behavioral dispositions or tendencies” (paragraph 1). D has to be wrong.
Since this question is asking about what the author would agree and disagree with, this is a main point question. I know that the author advocates functionalism: seeing mental events as caused by a confluence of internal and external factors. As a LEAST question, the wrong answers will involve multiple factors to explain why the climber accepts the rope, while the right answer will trend towards behaviorism and type-type identity theory and suggest that there is just a single reason why the climber would accept the rope.
A works as the right answer, because it gives just a single reason. However, LEAST questions can be tricky, and so it wouldn’t be the worst thing in the world to confirm my choice by eliminating the other answer choices.
B is in line with the author’s main point about how there are multiple factors that go into the mental state that leads the climber to accept the rope, so B has to be wrong.
C also mentions multiple factors, so C is wrong.
D also involves multiple factors—“processes” here is plural”—so D is also wrong, and A is correct.
Wow, this is an awkwardly worded question, in part because the situation in the question stem is so absurd. Whenever a question stem is confusing, it helps to try to read it in terms of the passage. The giraffe thing is weird, but it’s also just a belief, and I know that the passage is all about beliefs. I also know that the question is referring to functionalism, so that means I need to look at paragraphs 4 and 5. In paragraph 4, the author states that “Functionalism holds that mental events of a certain sort are to be identified as those underlying events, whatever they are, that function in a relevant way.” I still feel a little bit lost with regards to how to answer this question, which makes this a good opportunity to turn to the other resource I have: the answer choices. They tell me that I have 2 tasks: to determine if the giraffe situation is consistent with functionalism, and why that is or is not the case. I don’t see any obvious reason why the belief about giraffes wouldn’t be consistent with functionalism, since it doesn’t seem to challenge anything about functionalism itself, so that means I can restrict myself to A and B. So the question then becomes how the giraffe situation makes sense through the lens of functionalism, which I can do by testing A and B. Given how awkward this question is, I can usually trust that once I’m down to two answer choices, one will clearly echo the passage, while the other will be clearly wrong.
A seems to make some sense, since it restates the definition of functionalism from paragraph 4, but in a way that helps me to understand the giraffe situation. The belief about giraffes not wearing hats would only come into existence when a need for that belief arises: specifically, that would be something like someone asking if giraffes wear hats. Since that is a situation in which a mental event of a certain sort (the belief that giraffes don’t wear hats) resulting from an underlying event functioning in a relevant way (using general knowledge about giraffes and hats), A fits. To be sure of A, though, I should see if I can eliminate B.
B has to be wrong because it does not describe functionalism, which would hold that beliefs are mental events, not physical ones. B describes type-type identity theory, so it has to be wrong.
I can eliminate C because the right answer should say “Yes.”
Same with D.
While social construction is mentioned throughout the passage, the mention that most caught my attention starts paragraph 3 because it comes with an intensifier: “The idea that the biological self is socially constructed may seem outrageous.” The sentence implies an apparent opposition between biological and social construction, which I intuitively understand as the opposition between inside and outside the body. Since the rest of the passage is focused on explaining why this idea is not outrageous—I know this because the main idea, stated at the end of paragraph 5, is that “Nature and nurture stand in a relationship of reciprocity, not opposition”—I know I’m looking for an answer choice which involves external factors which could influence human biology that are mentioned in the passage.
“Interactions” is pretty vague, which immediately makes me suspicious of A. I also am not sure I fully grasp what it means to interact with peers “on personality traits.” Most importantly, though, there is nothing in A that even vaguely resembles the passage author’s examples, such as the last paragraph on language acquisition. That’s enough dissimilarity to make A incorrect.
B is way too abstract and not grounded in anything even vaguely resembling interactions with the world outside the body, so I can eliminate B.
C seems only tenuously related to what happens outside of the body through its mention of “experiences,” but C is made more attractive by how much it echoes what follows the quote from paragraph 3: “The idea that the biological self is socially constructed may seem outrageous. Admittedly, it is an extrapolation from data indicating that while the genes controlling embryonic development shape the infant brain, experience establishes the intricate neural connections underlying the brain’s functioning.” So even though biology (genes) shapes our brains, experience—which I can read as social construction, since it is constructed by interactions with the outside world—acts on our neuronal organization (neural connections). That’s enough similarity to make C seem right, but given how challenging this question is, I would feel better if I also eliminated D.
This is mentioned later in the passage, but with a lot of added context that is needed to connect D to the idea of social construction. To make D feel like the right answer, I actually first need to assume C. Since I need to make more logical leaps in D than in C, C has to be correct.
Mental illness is barely mentioned in the passage, so I should search for it. It is mentioned in paragraph 1: “consider the highly charged public discourse on whether mental illnesses are biological or psychological—that is, whether they are ‘no one’s fault’ or ‘caused’ by parents, spouses, the patients themselves, or social conditions.” Since the author exclusively discuses mental illness in terms of “fault” and blame, the right answer should involve not just what causes mental illness, but that sense of responsibility.
Severity or intensity isn’t mentioned in paragraph 1. The discussion of mental illness is described in ways that lack degrees; either one is mentally ill, or is not.
B works as a right answer, since it both implicitly involves fault or blame, and explicitly involves that sense of responsibility that the author mentions. Still, it’s not exactly what I predicted, so I would feel better quickly eliminating C and D.
C does refer to a cause of mental illness, but it’s so clinical that it doesn’t, by itself, involve the second half of the quote: where the author explains the import of the discussion over whether mental illness is biological or psychological. For me to read C as involving that discussion, I actually have to first assume something like B: that for it to be genetic is to indicate that it is someone’s fault. Since C doesn’t say that, I need more logical leaps to make C look correct than I do B, which makes B the better answer choice.
D is sort of the twin answer to C, insofar as it involves part, but not all of, the author’s discussion of mental illness. This time, it hones in on the part about fault in terms of morality, but doesn’t clearly involve the part about fault in terms of responsibility or cause. That makes D incorrect.
This question requires me to determine if there is a contradiction in the passage, and why that contradiction is either actual or just apparent. Intuitively, I have to believe that the assertions can be reconciled, simply because I know that kids definitely learn languages without explicit construction, and can be bad speakers of the language. But I need to use passage information to justify my answer, so looking at the last paragraph, which is where the discussion of language acquisition takes place, I can find support for my intuition: “the capacity to use grammar does not spring, like Minerva, fully formed from the head of Jove. Its acquisition and elaboration depend on social interaction. Whether a child acquires any language at all, let alone a specific language, is determined by the child’s linguistic community.” In other words, the situation in the question stem can be reconciled by the author’s argument. If children learn to speak through social interactions, then perhaps those older children just exist in a community where grammar is not strictly enforced. That’s enough to find the right answer.
A fits my prediction, since it gestures towards outside factors which could make those older children speak ungrammatically. A is good enough that I would choose it and move on.
B might be broadly true, but the author is talking about informal language learning, and so not learning in a formal setting: one in which someone teaches grammar. B is therefore incorrect.
I can eliminate C because the author does not suggest that kids generally learn a language from people who use grammar poorly. If that were the case, then all kids would be bad with grammar. C is also immediately wrong because I predicted that the first part of the answer should say “Yes.”
While the author would agree with D, the situation in the question stem does not obviously involve either learning disabilities or emotional disorders. I can also eliminate D because the first part of the answer choice should be “Yes.”
This is mentioned in paragraph 4, which was technical enough that I should probably reread that paragraph to answer this question: “We have learned that neurons and their processes overgrow luxuriantly in the course of development and that it is experience that selects the survivors.” That’s not quite enough for me to know what the author means by survivors, so the next sentence should help explain: “True, the basic ground plan is laid out in the genome, but the precise neuroanatomic details are specified by activity-dependent competition between presynaptic axons for common postsynaptic target neurons.” So the passage equates neurons surviving and “activity-dependent competition”: that is, competition based off of what is more active. I can thus deduce that neurons that can be classified as “survivors” are neurons that are especially active.
A is one way of saying that survivor neurons are active, since “participation” here implies activity. I would choose A and move on.
Living longer is one definition of survivor, but genes aren’t mentioned in paragraph 4. More importantly, B lacks any mention of “activity-dependent competition,” so it can’t be right.
This is tempting, because another synonym of survival is “adaptation.” However, C also lacks a clear reference to “activity-dependent competition,” since it doesn’t mention how frequently the neuron’s function is utilized. Without that key piece of the passage, C has to be wrong.
D has to be wrong because dendritic processes aren’t mentioned in paragraph 4.
Pregnant women are mentioned in paragraph 6, which I know from my initial reading of the passage contains the example of infants learning how to hear language in the womb. So I’m likely looking for an answer that involves talking.
A is an auditory activity, but not one that involves talking, so I can eliminate A.
B also doesn’t necessarily involve talking, so I can eliminate it.
C has the same problem.
D, though, definitely involves talking, which the passage claims the baby can overhear and so learn the language. I can therefore choose D with confidence and move on.
This is essentially a main idea question, since the passage is about the effects of competition between communities within a state and within the country. The author ends up advocating “increased cooperation” (paragraph 4), and notes that competition “can be destructive to the jurisdictions involved.” So I’m looking for an answer choice which suggests that there are serious downsides to such competition.
This is incorrect: in part because the passage does not discuss the benefits that private firms accrue from competition, but also because the author is pretty negative about this form of economic competition.
The passage does not mention anything about how to increase government efficiency, so B has to be wrong.
Given the author’s main argument in favor of greater cooperation between these subnational units, one could argue that the author would really like it if C were to happen. However, it is the lack of such coordination that actually encourages this contradiction, not the other way around (paragraph 1), so C has to be wrong.
D is what I predicted, and is the main idea of the passage, so I can choose it confidently and move on.
The passage brings up how difficult such cooperation is to encourage. For example, paragraph 6 describes how South Carolina could not foster cooperation between its counties. That implies that top-down mandates will not work: that it is only the leaders of these counties and cities who can overcome the “lingering rivalries” between each other and cooperate. This is reinforced by the last paragraph, which gives the example of local communities coming together in the Monongahela River Valley. There, “local leaders have journeyed across county and state boundaries to develop a coordinated agenda for economic revival.” So the right answer has to mention local leaders.
A is too analogous to the State Development Board in South Carolina: someone from above trying to impose power on those below. Since that board failed, national officials would also likely be unable to manage the many antagonisms that emerge between these local groups.
B is what I predicted, so I would choose it and move on.
Researchers aren’t really mentioned in the passage, and certainly not as viable encouragers of cooperation, so C has to be wrong.
Media representatives are also not really mentioned in the passage. It’s also just unclear how media representatives would handle the challenges to cooperation, which are largely political, since D doesn’t clearly suggest that these representatives have political power.
This is a nice and easy question. Since the Monongahela River Valley is an example of the author’s main idea, the right answer has to mention something about how the Valley is an example of what is possible when local jurisdictions cooperate.
A is the opposite of what the Monongahela River Valley suggests, since the Valley had to set aside competition in order to flourish.
B is also not what the author would suggest, since the example of the Monongahela River Valley is one of groups decreasing the level of competition between them.
C works, because it aligns with the author’s main idea: that cooperation works better than competition for these local jurisdictions.
No such grants are mentioned in the last paragraph, so D has to be wrong.
To answer this challenging question, I need to determine what the information in the question stem would weaken in the passage. It can’t be the author’s main idea per se, since it is an example of localities cooperating, so the right answer must mention one of the side or supporting arguments in the passage. What catches my eye in the question stem is that the counties began cooperating in a period of economic prosperity, which the author suggests is unlikely because competition is too easy and too natural. That means that the situation in the question stem weakens the claim in the last paragraph: “Economic conditions may ultimately serve as the catalyst for greater cooperation among jurisdictions.” The example of the Monongahela River Valley more specifically is meant to be an example of economic conditions—specifically, bad economic conditions—encouraging counties and local groups to cooperate. So the right answer will include something about how cooperation doesn’t have to only emerge during times of crisis.
The author never says that local jurisdictions never cooperate. There is an example of jurisdictions cooperating in the last paragraph. That makes A incorrect.
Nothing in the passage says that state law is the only way to get groups to cooperate. Again, the example of the Monongahela River Valley doesn’t involve any state laws, so B has to be incorrect.
This is correct. It is what I predicted, based off of the last paragraph. I can choose it and move on.
D also contradicts the last paragraph, which states that the leaders in the Monongahela River Valley journeyed across “state boundaries” to coordinate with other jurisdictions. D has to be incorrect.
The Great Lakes are mentioned in paragraph 4: “governors of the Great Lakes states have been unsuccessful in establishing a ‘no pirating’ pact within the region.” So I’m looking for an answer choice which I can derive from that quote.
A seems to fit, since that is exactly what the failure of the “no pirating” pact implies. I can therefore choose A and move on.
B has to be wrong because the Great Lakes states are depicted as failing to restrict economic competition between governments.
The discussion of the Great Lakes states focuses on the failure of the state governments to control economic competition between localities, but it does not suggest that local governments are any better. That makes C incorrect.
The national government isn’t even mentioned in paragraph 4, so I can eliminate this.
This question is asking about the author’s main idea: one that attempts to account for why so many seventeenth-century English scientists were Puritans. This is most clearly given in paragraph 4: “intellectually, their Puritanism did give them a special devotion to the honest observation of the world and a grave indifference to the authority of the past.” The right answer should resemble that quote.
The passage does mention that Charles II had to have had some problem with these Puritan scientists, but the passage never says anything about a lack of funding as hurting the scientific work of those scientists. A also lacks anything that resembles my prediction, so it must be incorrect.
If anything, the passage suggests the opposite in paragraph 4, when it states that the Puritans showed a “grave indifference to the authority of the past.” B is only a tempting answer choice if I bring in my outside knowledge of Puritans. Though “Puritans” is a term we often use now as a synonym for conservative, the Puritans of the time would have been considered radical in many ways. In any case, B has to be incorrect.
C also goes against the passage, which implies that the Puritan devotion to “honest observation of the world” played a role in the progress those Puritans helped achieve.
D closely resembles my prediction, so I should choose it and move on.
Given that the last question involved the main idea, I know that I’m looking for an answer choice which would suggest that the Puritans were somehow bad scientists. That said, I should also be ready for an answer choice that weakens an argument in support of the main idea as well.
This is more than good enough because it challenges the exact quote I drew from in the last paragraph. If the Puritans only accepted ideas that their religion allowed them to, that means there are theoretical limits to an “honest observation of the world” (paragraph 4). It would also be difficult to call them nonconformists, since A implies that the Puritans conformed to the demands of their religion. Since A directly opposes many of the key claims of the passage, A has to be correct.
B is actually in line with the passage logic, since it is one way of saying that the Puritans showcased “a grave indifference to the authority of the past.” B therefore cannot be correct.
ce depended at all on their leaders. C would have no discernible effect on the passage argument, so I can eliminate it.
Similarly, I don’t know what the Puritan pursuit of science would have to do with amassing wealth through commerce. More importantly, the passage mentions that many of the Puritan scientists came from wealthy merchant families (paragraph 4), so the mere presence of wealth or even the efforts to accrue it would not have any effect on the passage argument.
I know from my initial passage reading that while Charles II disliked the Puritans, he did endorse the Royal Society, which implies that he supported the Society’s mission of furthering scientific progress (rather than its people). So I’m looking for a piece of the passage which mentions that support. More importantly, though, I’m looking for an answer choice which tells me how Charles felt, which the wrong answers will fail to do.
A is a tricky answer choice because it explains why Cromwell may have felt some antagonism towards the society, but A doesn’t capture the most important reason why the author mentions Charles II: because Charles still supported the Royal Society in spite of what must have been at least some personal distrust of the society’s members. A is incorrect because it is not comprehensive enough.
B tells me that Charles had a reason to endorse the society, but it doesn’t tell me how he felt about it. So B has to be wrong.
C doesn’t tell me much about Charles’s attitude; Charles could have been happy that he had some supporters in the society, or unhappy that he had so few. Since I cannot easily determine how Charles felt about the society on the basis of C, I have to eliminate it.
D, though, does tell me how Charles felt about the society. On the one hand, it tells me that Charles supported its mission enough to approve its charter. On the other hand, since Charles could have done so much more, the mere approval of its charter suggests that Charles was dealing with “men whose political and religious views were distasteful to him” (paragraph 5). That’s more than enough to make D correct.
This is a difficult analogy question, since the answer choices are really dissimilar to the king and the Royal Society. To answer the question, I need to make sure I understand the relationship of the king and the Royal Society, generalize a description of that relationship, and then find an answer which fits that description. My work on the last question makes this relationship very clear: “Charles II can hardly have been enthusiastic about endorsing a society that was dominated by men whose political and religious views were distasteful to him” (paragraph 5). So I’m looking for a relationship of annoyance or distaste.
A fits what I’m looking for, because just as the king was annoyed by the Puritan scientists of the Royal Society, a dog is annoyed by fleas. However, analogy questions are difficult, and so it makes sense to eliminate the other answer choices just to confirm that I have the right answer.
A pterodactyl is an ancestor to birds, and is related to them. The king is not at all related to the Puritan scientists who helped found the Royal Society, so B has to be incorrect.
A lion eats gazelles, but I can’t figure out a way in which Charles II literally or figuratively hunted or consumed the Puritan scientists who founded the Royal Society. So C has to be incorrect.
A mouse is perhaps afraid of cats, or is prey to them. But while Charles II may have had good reason to be annoyed by the Puritans, the passage does not clearly suggest that the Puritans bullied him, or made Charles fearful in any way. That’s enough to make D incorrect, and A correct.
To answer this question, I need to check the descriptions of the two groups. Wallis’s group is mentioned in paragraph 1: Those who met with Wallis were interested in astronomy and geometry.” Boyle’s group is mentioned in paragraph 2: “Boyle was a philosopher, a physicist, and a chemist.” Neither group or person lists astrology—the belief that the positions of the stars can affect people’s lives—so I am looking for an answer choice which says that these papers were likely not presented in either group.
A fits what I’m looking for, so I can choose it and move on.
There is no good reason why those papers would be presented in Wallis’s group.
Same with Boyle.
And so it would obviously not make sense that they would be presented in both groups.
I know from my initial reading that the author claims that learned rules are part of “monitoring”: of speakers evaluating their phrases and editing them “to make minor changes or corrections in utterances which for the most part are initiated by acquired knowledge” (paragraph 2). The author then goes on to say in paragraph 3 that “even if rules are learned by students through explanation, drill, and practice, and even if they demonstrate that they can produce correct forms and syntax on grammar exams, such (cognitively based) knowledge is usually not very helpful in normal communicative situations.” For the author to then say that “oral proficiency in communication is not necessarily related to the ability to achieve high scores on standard grammar tests” is to imply that knowing the rules may help with grammar tests, but not with communication. So the right answer should say something about how learned rules are most helpful for a second language learner in grammar tests.
Underusers are defined in paragraph 4 as people “who rarely use their learned competence or perhaps those whose learned competence is low”: people, in other words, who aren’t all that in touch with the learned rules of a language. A has to be incorrect then, since the habits of an underuser wouldn’t necessarily involve using learned rules.
The author actually argues the opposite of B in the first paragraph: “learning activities [like learning the rules] are more limited in their usefulness to beginners.” That’s more than enough to make B incorrect.
C fits my prediction from paragraph 3, so I can choose C and move on.
Paragraph 3 states that learned grammar rules are “not very helpful in normal communicative situations,” so D has to be wrong.
Of the three kinds of users that Krashen mentions, I’m looking for one that is very aware of grammar rules. That has to be overusers, who are defined in paragraph 4 as spending “so much time and effort on correctness that it often seriously interferes with communication.”
Krashen doesn’t use the designation superusers. That term is coined by Carlos Yorio in the 4th paragraph, so A has to be wrong.
B works, since Krashen’s rating system is based off of how much speakers rely (or over-rely) on learned grammar rules.
Since optimal users are defined in paragraph 4 as “those who are able to monitor their speech and improve their level of grammatical accuracy, but not to such an extent that it interferes with smooth communication,” optimal users are unlikely to “frequently correct their own grammar while speaking,” which would be interruptive and make communication less smooth. C has to be wrong.
Since underusers are defined in paragraph 4 as having “low” competence when it comes to learned grammar, D cannot be right. Underusers would likely not be in touch enough with grammar rules to monitor as frequently as the people described in the question stem.
The author’s discussion of monitor theory mostly happens in paragraph 3 and 4, though the question stem’s focus on grammar tests helps me focus on paragraph 3. My work on prior questions tells me that monitor theory suggests that learners who are learning a language in a way that focuses heavily on grammar exams might not be getting better at actually communicating in that language. So what would trouble monitor theory would be an instance in which taking grammar tests was great for helping people to speak to other people in the foreign language they’re learning.
A is a great candidate for the right answer because it states what I predicted: that grammar tests are actually great indicators of oral proficiency, or the ability of people to actually communicate in the language they’re learning.
This is actually what the passage says happens, so B would strengthen the argument about monitor theory, not weaken it.
The reference to superusers in paragraph 5 implies that this is actually happening already: that language instructors probably already do well on grammar exams. That’s part of why they assign them, and think of them as integral to the language learning process. Since the author mentions C, and since the author isn’t obviously contradicting the passage argument here, C has to be incorrect. More importantly, C has to be wrong because monitor theory focuses on students, not instructors.
D is wrong for the same reasons C is wrong: because it focuses on instructors rather than students.
I know that the author distinguishes between learning grammar rules and acquiring language in “natural situations”: that is, learning language by communicating with others. So it stands to follow that since underusers are bad with grammar rules, the right answer has to involve learning to speak a language in a way that doesn’t involve grammar rules, and so instead involves “natural situations.”
A is perfect. This claim from paragraph 2 helps explain how underusers can become good communicators: by just learning to speak as children do, through actual conversations rather than through the formal work of the classroom.
B has to be wrong because it’s all about what doesn’t help language students, while the right answer has to explain how underusers can “achieve very high levels of communicative accuracy.”
C has to be wrong because underusers, by definition, don’t have a strong grasp of learned rules, so there is no way that they will be able to efficiently apply them.
Modifications, according to paragraph 2, are the result of monitoring, which underusers are bad at. So D can’t be right.
Since the passage is so down on learning grammar rules, I know that learning in something like an educational setting can’t be the right answer. The author instead seems to be advocating learning language in a “natural situation,” so I need to find an answer choice which involves informal verbal communication with other people.
This sounds like something one would do in a classroom, which is by definition a more formal learning environment. Since the author is so worried about how students learn in those environments, which focus so much on grammar rules, A can’t be right.
I know B is wrong by virtue of focusing on learning grammar. Paragraph 2 and 3 outline the problems with focusing on grammar: that most people can’t monitor well enough to use grammar rules in actual conversations.
C, though, is a good example of a “natural situation,” since it involves having an actual conversation in the second language. I would choose C and move on.
D also sounds a little like schooling, which makes me immediately suspicious of it. I can confirm that this is a problem in paragraph 3, since this sounds like learning “through explanation, drill, and practice.” The reason I would eliminate D, though, is that the primary way the passage refers to memory is when discussing the learning of grammar rules. Since the author is pretty negative about that approach for beginners, D has to be wrong.
Weird to have a math problem in the passage, but it’s really just a reading check. Since the answer choices mention percentages, I know to go to paragraph 2, where percentages are mentioned: “A quarter of the people born in seventeenth-century France died during their first year, another quarter died before the age of twenty, and a third quarter died by age forty-five.” Since the oldest child that the person in the question stem could have would be age 20, that means that the answer must be 50%, since that’s the result of adding the possibility that one’s children are part of the 25% with a chance of dying by the age of 1, and the 25% with a chance of dying before the age of 20.
These answer choices will only be tempting by virtue of twisting passage information. 10% is mentioned in the passage, but in reference to the percentage of the population that reached 60. The kids mentioned in the question stem definitely did not, so A is wrong.
This also twists the passage logic, since it’s likely a check to see if a reader will only pay attention to the part of paragraph 2 that states “another quarter died before the age of twenty.” However, that ignores the 25% who die by the age of 1, so B is incorrect.
C is what I predicted, and so I can choose it and move on.
The only way that one could get 75% from the passage is if one added up all the death percentages mentioned in the passage, but there’s absolutely no good reason to do that. The kids mentioned in the question stem don’t make it to 45, so D has to be wrong.
This is a key claim made in paragraph 1 that the author explains in the next two paragraphs. This also touches on the work I had to do to answer the last question, since seventeenth-century France is used as an example for the idea mentioned in the question stem. The author’s point must be that since so many people died when they were young, and so few people used to make it to old age in the past, then the association of death with old age has to do with the elimination of those causes of early deaths. The author confirms this in paragraph 3: “This startling increase [in average life expectancy for U.S. children] was due mainly to success in reducing infant, childhood, and maternal mortality,” all of which are deaths not associated with old age.
A fits my prediction, so I can choose it and move on.
Attachment isn’t mentioned in the passage at all, so I can eliminate B.
I can see why someone might choose C, since we tend to believe that long-term planning is generally a good thing, but there is nothing in the passage discussion contrasting deaths in the past with deaths today that mentions long-term planning, so I can safely eliminate C.
I also don’t remember respect being mentioned at all in the passage, so D is incorrect.
To answer this question, I need to understand how the quote in the question stem aligns with the passage. Most people—and especially Americans—have positive feelings about “pilgrims crossing the ocean to take up a new life.” The notion of a new life must be especially positive for the elderly, who are demographically defined by how old they are. So I imagine that the director of this retirement community is trying to tell these new arrivals that they will be happy there. That aligns with the last paragraph, where the author first mentions people who have concerns about retirement communities, but then states that “many gerontologists concluded that the elderly found in their segregated lives the advantages overtly and subliminally advertise in the real-estate brochures.” The right answer will therefore likely mention senior citizens being happy with moving into these communities.
A is wrong because there is nothing about ambivalence in the quote.
It’s not clear to me that immigrating pilgrims would be independent of their families; after all, wouldn’t many take them with them? More importantly, the passage never directly mentions anything about people in these communities moving there to be independent of their families, so I can safely eliminate B.
The passage is primarily focused on how these communities are great, and not on how hard life can get as an elderly person living outside of these communities, so C has to be wrong.
D is the last choice standing, and it fits what I’m looking for. Both the retirement community director and the author are pretty positive about these communities. And since part of being part of a community means being with people like you, D definitely qualifies as an assumption that the passage makes. I can choose it and move on.
This is a function question: one that is asking me about what role a piece of the argument plays in the larger argument. Having gone to this part of the passage before, I know that the reference to seventeenth-century France is meant to help explain the idea from paragraph 1 that “Until relatively recently, death had no more relation to old age than to any other period of life-in fact, it had less.” The right answer will be some abstractly-worded version of that.
This is a claim made later in the passage. The reference to France has nothing to do with changing residences, so it can’t clarify the relationship mentioned in A. That makes A incorrect.
It’s not clear how the reference to seventeenth-century France could suggest that America is not unique in terms of its demographic change. For B to make sense, seventeenth-century France would have to show similar demographic changes as those which the author claims are occurring in the U.S. But the author actually implies that what happened in the past is different from what is happening now. That makes B incorrect.
This is correct. C is the claim made at the end of paragraph 1 which both the mention of seventeenth-century France and twentieth-century America are meant to explain. C also fits the actual rhetorical effect of the reference to seventeenth-century France, since that reference is meant to show how old age used to be, in order to emphasize how much has changed in today’s America.
D is effectively the same answer as B, since for D to make sense, seventeenth-century France has to at least imply that people are living longer. That automatically makes D a bad answer choice, since both D and B cannot be right. But since the author uses the reference to France to show how things used to be, in order to use America as an example of how things are now, the author relies on there being a discontinuity between the examples of France and America: that the two together do not share a trend. D therefore has to be wrong.
This is a paradox question, so I need to use passage information to explain what feels like a logical problem. Paradoxes traditionally involve two terms that are in tension with each other. In this case, the two terms are in tension because they are phrased in such a way that I expect the life expectancy numbers to be similar to each other, when they are not. The quickest way to resolve a paradox like this is to determine how the two cases are different: what other differences between the two might account for this difference in life expectancy. The major difference, it seems, is that the first number is about Americans born in 1980, while the second number is about people who are sixty years old. That brings to mind the statistic about seventeenth-century France, and how few people in the past lived to old age. In other words, the second set is notable because it is talking about people who have already survived to old age; the people in that set were no longer at risk of all those early causes of death that made up the vast majority of human deaths until the late twentieth century. The implication here, then, is that old people in 1978 aren’t necessarily living longer lives: that the actual human lifespan hasn’t been expanded since 1900. So while life expectancy went up dramatically since 1900, because we got rid of many of the causes of early death, the lifespans of the elderly did not. That has to be enough to explain the paradox.
A doesn’t make sense, since the information in the question stem doesn’t mention any effects related to more people getting older. It’s also just hard to see how more people getting older would cause the numerical discrepancy that the question stem asks about. A is therefore wrong.
B is meant to be attractive by virtue of invoking how the life expectancy associated with 1980 is bigger than the one associated with 1978. But B has to be wrong because it does not clearly involve the other half of the answer choice: the discrepancy in the ages of those whose statistics are mentioned. B is therefore incorrect.
It’s possible that one can read into the mention of sixty year old white men the idea that the bigger number mentioned in the first half of the question stem has something to do with women. But the passage doesn’t really focus on women living longer than men do, so I’m reluctant to pick C. Hopefully D is a clear right answer.
Luckily, D is right because it matches my prediction. The paradox in the question stem is only a seeming paradox because of the way the question plays with life expectancy and longevity. This aligns with the author’s claim that the increase in average life expectancy “was due mainly to success in reducing infant, childhood, and maternal mortality,” with only a small increase in longevity (paragraph 3).